Missed Questions- MIXED SUBJECT

Lakukan tugas rumah & ujian kamu dengan baik sekarang menggunakan Quizwiz!

An employee decided to kill his boss, after she told him that he would be fired if his work did not improve. The employee knew his boss was scheduled to go on a business trip on Monday morning. on Sunday morning, the employee went tot he company parking garage and put a bomb in the company car that the boss usually drove. The bomb was wired to go off when the car engine started. The employee then left town. At 5 am Monday, the employee, after driving all night, was overcome with remorse and had a change of heart. He called the security officer on duty at the company and told him about the bomb. The security officer said he would take care of the matter. An hour later, the officer put a note on the boss' desk telling her of the message. He then looked at the car but could not see any signs of a bomb. He printed a sign saying "DO NOT USE THIS CAR," put it on the windshield, and went to call the police. Before the police arrived, a company vice-president (not the boss) got into the car and started the engine. The bomb went off, killing her. The jurisdiction defines murder in the first degree as any homicide committed with premeditation and deliberation or any murder in the commission of a common-law felony. Second-degree murder is defined as all other murder at common law. The employee is guilty of (A) murder in the first degree, because, with premeditation and deliberation, he killed whoever would start the car. (B) murder in the second degree, because he had no intention of killing the company vice president. (C) manslaughter, because at the time of the explosion, he had no intent to kill, and the death of the company vice-president was in part the fault of the security officer. (D) only attempted murder of the boss, because the death of the company vice-president was the result of the security officer's negligence.

Answer Choice A is the best choice. because the employee acted with the intent to kill whoever started the car, and then killed that person. This is basically a problem involving "concurrence" -- the examiners are thinking you'll reason. the requisite intent no longer existed at the time of the explosion, therefore the requirement of concurrence between act and mental state has not been satisfied. It is true that concurrence is required. But in the case of a crime defined in terms of a particular result (like murder), the requisite concurrence is between mental state and act, not between mental state and result. In other words, at the moment a defendant takes the act that brings about the result, the defendant must be actuated by the appropriate intent; it doesn't matter whether the defendant still has that intent when the result finally occurs. Here, the "act" was the setting of the bomb. When the employee planted the bomb, he was actuated by an intent to kill his boss. The fact that before the bomb went off (producing death as a result) he had changed his mind is irrelevant. There is a further element to worry about: the fact that an unintended victim (the company vice-president), not the intended victim (the boss) was killed. But this makes no difference either, under the familiar doctrine of transferred intent, by which, if the type of harm intended is the type that results, the fact that a different victim ended up suffering that harm is irrelevant. Lastly, the fact that the security guard had opportunity to avoid the harm but failed to do so is irrelevant -- the employee intended to bring about a death by bomb, and his act was the but-for cause of that death by bomb, so the fact that some other actor failed in a chance to avoid the harm makes no difference. And that's true even if the failure by the other person amounted to negligence.

While the corporate developer of a condominium was properly in control over the condominium association, the association entered into a contract with a subsidiary of the developer to provide maintenance services to the condominium for 10 years. The contract commits the association to pay a modest, but not unconscionable, amount in excess of the fair market value of such services. The contract also provided a substantial penalty in the form of a valid liquidated damages provision for cancellation of the contract prior to its termination. Once control of the association is turned over to the unit owners other than the developer, can the association cancel this contract without penalty? A. Yes, because the maintenance contract was made with an affiliate of the developer. B. Yes, because the contract price for the maintenance services exceeds the fair market price. C. No, because the developer was properly in control of the condominium association when the association entered into the contract. D. No, because the contract price for the maintenance services is not unconscionable.

Answer Choice A. Once a developer has relinquished control, the association has the power to terminate without penalty any contract or lease between the association and the developer, or an affiliate of the developer. Consequently, the association may cancel this contract without incurring a penalty. Answer choice B is incorrect because, although generally the reason for cancelling a contract made with a developer or an affiliate is that it is not fair to the association, the association has the right to cancel any contract made with the developer or an affiliate. Answer choice C is incorrect. At the time that the contract was entered into by the developer, by virtue of his control over the association, he had the power to cause the association to enter into the contract. Once control of the association was turned over to the unit owners, who are not the developer, they may cancel such contract without penalty. Answer choice D is incorrect. The association may cancel without penalty a contract made with an independent third party that was unconscionable to the members other than the developer at the time the contract was entered into. However, the association has the power to terminate any contract made while the developer was in control of the association, regardless of its fairness.

A widower devised his fee simple interest in his residence as follows: "to my daughter for life, then to my oldest grandchild who survives her." At the time of the widower's death, he was survived by his only two children, a son and a daughter, and by one grandchild, his daughter's son. A short time later, the daughter together with her son entered into a contract to sell the residence in fee simple to a buyer. The applicable jurisdiction continues to follow the common law Rule Against Perpetuities, but has abrogated the rule in Shelley's Case. At the closing, the buyer refused to purchase the residence. Can the sellers compel the buyer to do so? A. No, because the son cannot transfer his contingent remainder interest. B. No, because the sellers have breached the covenant of marketable title. C. Yes, because any other grandchildren of the widower have no interest in the residence due to the Rule Against Perpetuities. D. Yes, because the jurisdiction has abrogated the rule in Shelley's Case.

Answer Choice B. Absent contrary language, an implied covenant of marketable title is part of a contract for the sale of real property. Marketable title is a title that is free from an unreasonable risk of litigation. Among the defects in title that render the title unmarketable are future interests held by persons who have not agreed to the transfer. Here, it cannot presently be determined with reasonable certainty who, if anyone, will be the widower's oldest grandchild to survive the widower's daughter. Although that person may be the daughter's son, because both the widower's son and daughter could have additional children and the daughter's son, regardless of his age, could die before the daughter, the sellers are unable to convey marketable title to the buyer. Answer choice A is incorrect. The daughter's son does have a remainder because his future interest follows a possessory estate of known fixed duration (i.e., his mother's life estate) that was created in the same conveyance (i.e., his grandfather's will), and that remainder is contingent because it is subject to an express condition precedent to his taking the residence (i.e., he must survive his mother). Although, at common law, a contingent remainder could not be transferred inter vivos, most jurisdictions today permit such a transfer. Moreover, if the applicable jurisdiction does not permit such a transfer, then the widower's daughter, acting alone, can only transfer her life estate interest in the residence to the buyer. Answer choice C is incorrect. Under the Rule Against Perpetuities (RAP), specific future interests are valid only if they must vest or fail by the end of a life in being plus 21 years. Here, the widower's daughter is a life in being at the time of the widower's death. It will be known at her death if there is a grandchild who outlives her and, if there is more than one, the grandchild who is the oldest. Answer choice D is incorrect. Although the jurisdiction has abrogated the rule in Shelley's Case, this rule, under which a present interest in the grantee and contingent remainders in the grantee's heirs became a fee simple interest in the grantee, does not apply in this case. Here, the remainder is in a specific person, not generally in the heirs of the widower's daughter, even though the specific person could be an heir of the daughter.

State X recently enacted a statute that required any sushi-grade fish sold in State X to sushi restaurants to be sourced from a State X fishmonger located no further than 50 miles from any sushi restaurant located in State X. The purpose of the regulation was to ensure that fish that was typically consumed in a raw form in sushi restaurants maintained its quality and freshness during delivery. A fishmonger located in State Y, which borders State X, is located 25 miles away from a group of sushi restaurants in State X that used to buy their fish from him prior to the enactment of the statute. If the State Y fishmonger challenges the constitutionality of the statute, will he prevail? A. Yes, because the statute violates the Privileges or Immunities Clause. B. Yes, because the statute unduly burdens interstate commerce. C. No, because the statute protects an important local interest. D. No, because State X is acting as a market participant.

Answer Choice B. The Dormant Commerce Clause limits the power of the states to legislate in ways that impact interstate commerce. If Congress has not enacted legislation in a particular area of interstate commerce, then the states are free to regulate, so long as the state or local action does not (i) discriminate against out-of-state commerce, (ii) unduly burden interstate commerce, or (iii) regulate extraterritorial (wholly out-of-state) activity. Here, the statute discriminates against out-of-state fishmongers by requiring sushi restaurants in State X to purchase fish only from State X fishmongers located within 50 miles of the restaurants. If a state or local regulation, on its face or in practice, is discriminatory, then the regulation may be upheld if the state or local government can establish an important local interest is being served and no other nondiscriminatory means are available to achieve that purpose. Here, the purpose of the regulation was to ensure that fish that was typically consumed in a raw form in sushi restaurants maintained its quality and freshness during delivery. Thus, the statute likely serves an important local interest. However, there are other nondiscriminatory means available to achieve that interest. The State Y fishmonger is located 25 miles away from sushi restaurants located in State X. Thus, the purpose of the statute would be accomplished if he delivered his fish to the State X restaurants. Therefore, the statute places an undue burden on interstate commerce and should be struck down. Answer choice A is incorrect because the Fourteenth Amendment provides that "[n]o state shall make or enforce any law which shall abridge the privileges or immunities of citizens of the United States." Although this clause has been interpreted to protect a right to interstate travel, it does not apply to cases of discrimination based on the sale of fish across state borders. Answer choice C is incorrect. Although the statute likely serves an important local interest, other non-discriminatory means of achieving the same important health and food safety interests are available. Answer choice D is incorrect. A state may behave in a discriminatory fashion if it is acting as a market participant (buyer or seller), as opposed to a market regulator. Here, State X is acting as a market regulator, not a market participant. Therefore, this exception does not apply.

At about 5:00 a.m., firefighters responded to a house fire. The fire was brought under control in about an hour and completely extinguished approximately two hours later. In the early afternoon, a city fire inspector came to the house with an administrative warrant. The warrant authorized the inspector to search for the cause of the fire and seize items related to it. The inspector first searched the basement of the house, finding a barely recognizable electric curling iron plugged into a partially melted timer, which was set to turn on at about 4:00 a.m. In addition, the inspector found the iron in a large soot blackened tub. Nearby were several empty gallon containers labeled "turpentine." The inspector seized the iron, timer, tub and empty turpentine containers, believing that they constituted evidence necessary to establish the cause of the fire. While the ground level of the house was almost completely destroyed by the fire, the second level was in much better condition. On the second floor, the inspector noticed that there were several empty frames, their pictures apparently having been removed, hanging on the walls. Also the inspector noted that none of the three upstairs bedrooms contained electronic equipment, such as televisions or computers, despite the presence of empty power strips plugged into outlets in these rooms. The inspector seized the empty frames and power strips, believing that they constituted evidence the homeowner had deliberately set the fire. The family who lived in the house was away on vacation at the time of the fire and did not return until the following day. The homeowner was charged with arson. Can the homeowner successfully object to the introduction into evidence of the items seized by the inspector from the second floor of the house? A. No, because the items were seized by a fire inspector rather than a police officer. B. No, because the search was conducted under exigent circumstances. C. No, because the search was conducted pursuant to an administrative warrant. D. Yes, because the seizure of these items exceed the scope of the warrant.

Answer Choice D. Although the warrant authorized the fire inspector to search for the cause of the fire, the fire inspector determined the cause of the fire in his search of the basement. As a result, his search of the second floor of the house exceeded the scope of the administrative warrant. On the second floor, he was searching for and found evidence related to whether the homeowner had deliberately set the fire, i.e., whether the homeowner was guilty of arson, rather than the cause of the fire. Consequently, answer choice C is incorrect. Answer choice A is incorrect because the Fourth Amendment protection against unreasonable searches applies to a search conducted by a governmental agent. It does not require that the agent be a police officer. Answer choice B is incorrect because, since the fire inspector conducted the search pursuant to a warrant, there is no need to justify the search under an exception, such as exigent circumstances, to the warrant requirement. In addition, although a search for the cause of the fire conducted within a reasonable time after the fire was extinguished may be justified under the exigent circumstances exception, this exception does not apply to a search for evidence that arson was the cause of the fire.

The chief financial officer (CFO) of a large publicly traded corporation owned stock in an unaffiliated company. The CFO, hoping to convince his friend to buy his stock in the company, told the friend that the value of the company's stock would increase dramatically over the next three years. Neither individual possessed specific knowledge about the financial health of the company. However, based on the CFO's statement, the friend reasonably believed that the CFO had such knowledge. The friend did not attempt to investigate the company's finances, but instead, relying on the CFO's statement, entered into a contract with the CFO for the friend to purchase half of the CFO's stock in the company. The following day the company filed for bankruptcy. Can the friend rescind the contract due to misrepresentation? A. No, because the CFO's statement was merely his opinion as to the future profitability of the company. B. No, because the friend failed to investigate the financial health of the company. C. Yes, because the CFO's statement regarding the future profitability of the company was material. D. Yes, because the CFO's statement constituted a false assertion about the company's present financial condition.

Answer Choice D. A misrepresentation is an untrue assertion of fact. A fraudulent misrepresentation can render a contract voidable by the adversely affected party if (i) the misrepresentation is fraudulent (i.e., a false assertion of fact made knowingly or recklessly without knowledge of its truth, and with the intent to mislead the other party), (ii) the misrepresentation induced assent to the contract, and (iii) the adversely affected party justifiably relied on the misrepresentation. A statement as to a future event is generally an opinion rather than a statement of fact. However, when a statement about a future event implies that it is based on facts unknown to the listener, the listener may be justified in believing that the speaker knows of facts that justify the opinion, or at least that the speaker does not know of any facts that are incompatible with the opinion. Here, the CFO's reckless statement regarding the future value of the company's stock implied that the CFO knew that the company's present financial condition was sound. Accordingly, answer choice A is incorrect. As the friend relied on the CFO's fraudulent misrepresentation, the friend can rescind the contract. Answer choice B is incorrect. Although a party's reliance on a misrepresentation must be justified, a party's failure to exercise reasonable care to avoid the mistake caused by the misrepresentation is not sufficient to make the party's reliance unjustified. A party's reliance is unjustified only in the extreme case when, based on all of the facts and circumstances, the party has failed to act in good faith and in accordance with reasonable standards of fair dealing. Here, although the friend did not investigate the financial health of the company, this failure does not make the friend's reliance on the CFO's misrepresentation unjustified. Answer choice C is incorrect. A misrepresentation is material when the information would cause a reasonable person to agree, or the person making the misrepresentation knows the information would cause this particular person to agree. Although the CFO's misrepresentation was likely material, because the CFO's misrepresentation was fraudulent, the friend can seek to rescind the contract regardless of whether it was material.

At 11:00 pm, a husband and his wife were accosted in the entrance to their apartment building by the defendant, who was armed as well as masked. The defendant ordered the couple to take him into their apartment. After they entered the apartment, the defendant forced the wife to bind and gag her husband and then to open a safe which contained a diamond necklace. The defendant then tied her up and fled with the necklace. He was apprehended by apartment building security guards. Before the guards could return to the apartment, but after the defendant was arrested, the husband, straining to free himself, suffered a massive heart attack and died. The defendant is guilty of (A) burglary, robbery, and murder (B) robbery and murder only (C) burglary and robbery only (D) robbery only

Answer Choice A. Because the defendant can be found guilty of all three crimes. Burglary is (1) the breaking, (2) and entering, (3) of the dwelling, (4) of another, (5) at night time, (6) with the intent to commit a felony within.

An undercover police detective told a local drug dealer that she wanted to buy cocaine, but that she needed time to raise the necessary funds. The drug dealer said that he needed time to get the cocaine. They agreed to meet again in ten days. An hour later, without a warrant, other officers forcibly entered the drug dealer's apartment and arrested him for attempted possession of a controlled substance. If the drug dealer is prosecuted in a common-law jurisdiction for attempted possession of cocaine, should he be convicted? (A) No, because he had not taken sufficient acts toward commission of the crime. (B) No, because he was illegally arrested. (C) Yes, because by objective standards an agreement between them had occurred. (D) Yes, because his intention to obtain the cocaine was unequivocally expressed.

Answer Choice A is the best response. Because the defendant must go beyond mere preparation in order to be convicted for attempted possession. The common law of attempt required that the defendant commit some act (beyond mere "preparation") toward bringing about the intended crime. Here, the drug dealer took no act, much less any act that would qualify at common law, before obtaining the cocaine. Indeed, the drug dealer likely would not be guilty of attempt even under the Model Penal Code's broadened standards because there was no "substantial step" toward commission of the crime. Thus, under these facts, solely stating that you need more time and agreeing to meet again will not qualify as acts b5riging about the intended crime.

The owner of a car was severely injured and her car was destroyed when she became distracted by a display on an electronic billboard and lost control of her car. The car owner brought a negligence action based on diversity jurisdiction in federal district court against the owner of the billboard, as well as the advertising firm that created the display and the manufacturing company that sold the product displayed. The manufacturing company filed a cross-claim against the advertising firm based on an indemnification clause in their contract. Instead of filing an answer, the advertising firm served a motion to dismiss the cross-claim for failure to state a claim upon which relief can be granted. An affidavit regarding the oral waiver of the indemnification clause was attached to the motion. Before the court takes any action with regard to the advertising firm's motion, can the manufacturing company voluntarily withdraw the cross-claim without the approval of the court or the consent of the parties? A. Yes, because the advertising firm has not served an answer to the cross-claim B. Yes, because a cross-claim is never compulsory. C. No, because the advertising firm's motion to dismiss was converted into a motion for summary judgment. D. No, because the advertising firm served a responsive pleading.

Answer Choice A. A party may voluntarily dismiss a cross-claim (or a counterclaim or third-party claim) without the approval of the court or the consent of the parties before a responsive pleading is served, or if there is no responsive pleading, before evidence is introduced at a hearing or trial. The service of a summary judgment motion by a co-party with respect to a cross-claim (or counterclaim or third-party claim) does not cut off a party's ability to voluntarily dismiss the claim without court approval or the consent of the other parties. This occurs only after evidence is introduced at the court hearing on the motion. Here, although the motion filed by the advertising firm was a motion to dismiss for failure to state a claim upon which relief can be granted, the attachment of an affidavit presenting evidence beyond the pleadings converted the motion into a summary judgment motion. Nevertheless, because the court had not held a hearing on this motion at which evidence was presented, the manufacturing company can voluntarily withdraw the cross-claim without court approval or the consent of the parties. Accordingly, answer choice C is incorrect. Answer choice B is incorrect. While it is true that a cross-claim is never compulsory, this fact does not affect the ability of the party who files a cross-claim to voluntarily withdraw that claim without court approval or consent of the other parties. Answer choice D is incorrect. As explained with regard to answer choice A, the manufacturing company can voluntarily withdraw the counterclaim without court approval or consent of the parties. In addition, the facts indicate that the advertising firm did not file a responsive pleading (an answer), but instead filed a motion to dismiss.

Members of a religious organization, who were citizens of State A, chartered a bus to attend a conference. The bus, which was owned and operated by a company incorporated in State B and with its principal place of business in the same state, was involved in an accident in State C. The members of the organization who were injured in the accident filed a negligence action in the federal district court for State C. The bus company seeks to preclude the federal court for State C from hearing the action. Which of the following actions is most likely to achieve this result? A. A motion to transfer the case to a federal court for State B. B. A motion to dismiss for lack of personal jurisdiction over the bus company. C. A motion to dismiss for improper venue. D. A motion to dismiss based on the forum non conveniens doctrine.

Answer Choice A. Although as discussed in the explanation for answer choice C venue is proper in State C, the federal district court for State C can nevertheless transfer any civil action to any other district or division where it might have been brought for the convenience of parties and witnesses and in the interest of justice. Since venue in a federal civil action is proper in a judicial district in which any defendant resides, if all defendants reside in the same state in which the district is located, venue is also proper in State B. Consequently, the motion to transfer the case to a federal court in State B is the alternative most likely to preclude the federal court for State C from hearing the action. Answer choice B is incorrect. Although the bus company is not a citizen of the forum state (State C) but of State B, where it is incorporated and has its principal place of business, the federal court for State C would have personal jurisdiction over the bus company, assuming that the bus company was properly served with process, on the basis of specific jurisdiction because the cause of the action arises out of the bus company's conduct in State C (i.e., the accident). Answer choice C is incorrect. Venue in a federal civil action is proper in a judicial district in which a "substantial part of the events or omissions" on which the claim is based occurred, or where a "substantial part of the property" that is the subject of the action is located. Here, since the accident that gave rise to the plaintiff's injuries occurred in State C, venue is proper in State C. Answer choice D is incorrect. The common-law forum non conveniens doctrine allows a court to dismiss an action—even if personal jurisdiction and venue are otherwise proper—if the court finds that the forum would be too inconvenient for parties and witnesses and that another, more convenient, venue is available. To convince a court to invoke this doctrine, the defendant ordinarily bears a heavy burden to displace the plaintiff's chosen forum. In light of the availability of a statutory based transfer of the action to another federal court, the court is unlikely to dismiss the action in reliance on the forum non conveniens doctrine.

Three siblings, two brothers and a sister, inherited land as equal tenants in common from their mother. The property was subject to a mortgage that contained an acceleration clause, which provided that the entire outstanding balance of the mortgage loan was due upon default. None of the siblings made the mortgage payments as they became due and the mortgage fell into default. The mortgagee foreclosed on the mortgage. At the foreclosure sale, one of the siblings, the sister, purchased the land, paying 45 percent of the land's fair market value. Can either of her two brothers reclaim his interest in the land? A. Yes, if he pays his sister one-third of the amount she paid to redeem the land. B. Yes, if he pays his sister one-third of the fair market value of the land. C. No, because co-tenants do not owe each other fiduciary duties. D. No, because their interests were eliminated through the foreclosure sale.

Answer Choice A. Although co-tenants owe a duty of fair dealing to each other, co-tenants generally do not owe fiduciary duties to each other. However, a fiduciary obligation can be imposed on co-tenants who jointly purchase the property in reliance on each other or acquire their interests at the same time from a common source, such as by gift, will, or inheritance. Typically, these co-tenants will be related or in a confidential relationship. The primary situation in which such a co-tenant is found to have a fiduciary obligation arises when the property is sold at a tax or mortgage foreclosure sale and a co-tenant acquires the property. In such a situation, the other co-tenants have the right to reacquire their original interests by paying their due contributions within a reasonable time. Here, the co-tenant purchaser, the sister, can be compelled to permit the other former co-tenants the opportunity to reacquire their interests in the property by paying their share of the acquisition costs. Answer choice B is incorrect because the former co-tenants are not required to repurchase their interests based on the fair market value of the property; rather, they can repurchase their interests based on the amount that the purchasing co-tenant, the sister, paid to acquire the property. Answer choice C is incorrect. Although generally a co-tenant does not owe fiduciary duties to another co-tenant, a fiduciary obligation can be imposed on co-tenants who jointly purchase the property in reliance on each other or acquire their interests at the same time from a common source, such as by gift, will, or inheritance. The primary situation in which such a co-tenant is found to have a fiduciary obligation arises when the property is sold at a tax or mortgage foreclosure sale and a co-tenant acquires the property. In such a situation, the other co-tenants have the right to reacquire their original interests by paying their due contributions within a reasonable time. Answer choice D is incorrect because, although the buyer at a foreclosure sale generally takes free of the mortgagor or the mortgagor's successors in interest, here the sister, as a co-tenant, can be compelled to permit the other former co-tenants the opportunity to reacquire their interests in the property by paying their share of the acquisition costs.

In state court, an individual was found guilty of misdemeanor possession of a controlled substance, and the court imposed a sentence of six months' imprisonment. The execution of the sentence was suspended and she was placed on probation for six months. The individual violated a condition of her probation by failing to attend a treatment program. The individual admitted her violation at a preliminary probation revocation hearing, but at her final probation revocation hearing the individual made the necessary proffer of indigence and a timely request for the appointment of counsel, which the court denied. At this final hearing, the court, as permitted by state law, revoked her probation and executed the sentence of six months for violation of the terms of her probation. Did the individual have an absolute constitutional right to the appointment of counsel at the final probation hearing? A. No, because execution of the defendant's sentence was triggered by revocation of her probation. B. No, because the court's sentence did not exceed six months' imprisonment. C. Yes, because the court's action resulted in her incarceration. D. Yes, because an indigent defendant has the right to the appointment of counsel.

Answer Choice A. An offender does not have an absolute constitutional right to counsel at a probation revocation hearing when an already-imposed sentence is executed as a result of the revocation of probation. Instead, an offender only has such a right if it is necessary for a fair trial. Accordingly, the individual did not have an absolute constitutional right to the appointment of counsel. Answer choice B is incorrect because, while the right to a jury trial is keyed to whether the offense carries a punishment of imprisonment for more than six months, the right to counsel is not subject to this time period limitation. In general, a defendant's right to counsel applies in any trial at which the defendant is sentenced to incarceration. However, a defendant does not have an absolute constitutional right to counsel at a probation revocation hearing. Answer choice C is incorrect because her sentence had already been imposed, so she had only a limited, not an absolute, constitutional right to the appointment of counsel. Answer choice D is incorrect because, while an indigent defendant who has a right to counsel does have the right to the appointment of counsel, an offender does not have an absolute constitutional right to counsel at a probation revocation hearing.

A man and his friend were watching a televised football game at the man's home. Upset by a penalty called by the referee, the friend threw a bottle of beer at the man'[s television, breaking the screen. Enraged, the man picked up a near by hammer and hit the friend on the head with it. The friend died from the blow. The crimes below are listed in descending order of seriousness. In a jurisdiction that follows common law principles, what is the most serious crime of which the man could properly be convicted? (A) Murder (B) Voluntary manslaughter (C) Involuntary Manslaughter (D) Assault

Answer Choice A. Because one of the mental states that suffices for murder is an intent to do serious bodily harm. At common law, a number of mental states can suffice for murder. One of these mental states is the intent to do serious bodily injury. The intent to serious bodily injury like the intent to kill, can be inferred from the defendant's conduct in the light of the surrounding circumstances. The defendant's choice of weapon, and the nature of the physical attack on the victim, are circumstances that can lead a reasonably jury to infer an intent to seriously injure the victim. Here, the man's choice of a hammer, and his swinging the hammer at his friend's head rather than at some other point on the body, would reasonably justify a reasonable jury in concluding be4yond a reasonable doubt that the man intended to cause at least serious bodily injury to his friend. Since the friend's death directly resulted from the man's conduct, all the requirements for murder of the intent-to-do-serious-bodily-injury variety are present. (It's possible -- though not likely-- that the jury might accept the partial defense of reasonable provocation, which would downgrade the crime to voluntary manslaughter. But a reasonable jury could certainly reject the provocation defense, in which case a murder conviction would be most appropriate, and you're asked to specify the most serious crime of which the defendant "could properly be convicted".)

A store clerk had been working at a convenience store for four years when a new owner took over. On the second day the new owner was at the store, she fired the clerk. The clerk was so angry that he decided to kill the new owner. However, he knew the store was frequented by police officers, and he was afraid of being seen, caught, arrested, or even shot. When he arrived home, the clerk told his girlfriend, whom he knew to be vengeful and easily provoked to violence, that the new owner had made sexual advances towards him. The girlfriend became extremely angry and said, "I want to kill her." The clerk said, "I would, too, if I were you. I'd kill her so fast she wouldn't know what hit her." The girlfriend grabbed a large machete from the garage and headed to the store. As she was walking into the store, a police officer outside the store saw her and, believing she was going to kill someone, shot her. The girlfriend died from the gunshot. If the state charges the clerk with the murder of the girlfriend, is he likely to be convicted? A. Yes, because he acted with a depraved heart. B. Yes, because his intent to kill the storeowner transferred to the girlfriend. C. No, because he never intended his girlfriend to be shot. D. No, because the Redline doctrine prevents his conviction for the girlfriend's death.

Answer Choice A. Common-law murder is the unlawful killing of another human being committed with malice aforethought. When the store clerk purposefully inflamed his girlfriend in an effort to have her kill the new owner, his reckless indifference to the value of human life (depraved heart) resulted in his girlfriend's death. The clerk knew that the store was frequented by police officers but nonetheless enticed his girlfriend to go to the store with a large machete to kill the new owner. As a result, he will likely be convicted of the girlfriend's murder. Answer choice B is incorrect because the doctrine of transferred intent is inapplicable. Here, the girlfriend did not die from the clerk's misdirected attempt to kill the new owner. She died because the police officer committed a justifiable homicide to prevent her from killing someone. Answer choice C is incorrect because the store clerk need not have intended to kill his girlfriend to have the requisite intent for murder—a reckless indifference to an unjustifiably high risk to human life is sufficient to meet the requirement of malice aforethought. Answer choice D is incorrect. Under the Redline doctrine, a defendant is generally not guilty of felony murder when a victim or a police officer, acting in self-defense or trying to prevent the escape of the defendant or his co-felon, kills the co-felon. However, this is only applicable as a defense to felony murder for the death of a co-felon during a felony. It does not bar conviction of the clerk for a depraved heart killing.

The owner of land executed an instrument that purported to convey the land to a transferee. The instrument, which contained all the necessary elements of a deed, contained the following statement, "This transfer is made for the sum of $1.00 and other good and valuable consideration." The transferee promptly and properly recorded the instrument. Subsequently, the owner challenged the validity of the deed based on lack of consideration. The owner can prove that he did not receive any cash or other consideration from the transferee. Should the deed be set aside? A. No, because consideration is not required to effect a transfer of land. B. No, because the instrument has been recorded. C. Yes, because parol evidence may be introduced to contradict the recitation in the deed of the receipt of consideration. D. Yes, because the act of recording does not validate an otherwise void deed.

Answer Choice A. Consideration is not required to effect the conveyance of real property. Here, the instrument contained all the necessary elements of a valid deed. Consequently, it operates to effect the conveyance of the land regardless of whether or not the recited consideration was paid. Answer choice B is incorrect because, although the instrument was recorded, recording would not validate an otherwise void deed. However, the deed in question was not void, even though the recited consideration was not paid, because it contained all the necessary elements of a valid deed. Answer choice C is incorrect. Consideration is not required for a deed to be valid and thus there is no need to introduce parol evidence regarding the receipt of consideration. Answer choice D is incorrect. Although recording does not validate a void deed, the instrument in question was not void because consideration is not required in order to convey real property.

Two rock climbers, a man and a woman, decided to climb a rock outcropping over a hiking trail. A few hours later, a park ranger discovered the two rock climbers and a hiker lying injured and unconscious on the trail. It looked like the rock climbers, who were still tethered together, had fallen on the hiker. The park ranger called for medical assistance and, with the other rangers, was able to bring all three injured parties down the mountain to an ambulance. While in the ambulance, the woman regained consciousness. She told the ambulance technician that she and the man had ingested hallucinogenic drugs before their climb, and that as she remembered the incident, the man had unexpectedly jumped when they reached the top of the outcropping. The hiker has sued the other man for negligently inflicting the hiker's injuries. Although the woman is available to testify, the hiker wants to call the ambulance technician to testify as to the woman's account of the fall. May the ambulance technician testify as to the woman's statements? A. No, because the statements are hearsay not within any exception. B. No, because the woman is available to testify. C. Yes, because the statements could expose the woman to legal liability. D. Yes, because the statements were made to the ambulance technician.

Answer Choice A. Hearsay is a statement that the declarant makes at a time other than while testifying at the current trial or hearing (i.e., an out-of-court statement) that is offered to prove the truth of the matter asserted. Hearsay evidence generally is inadmissible unless it falls within an exception or exclusion set out in the Federal Rules. Here, the woman's statements to the ambulance technician are hearsay. Accordingly, the statements are only admissible if they fall within an exclusion or exception to the hearsay rule. The statements, while relevant, are not an admission by an opposing party because the woman is not a party in the hiker's action. The statements do not fall within any other exclusion. In addition, no exceptions to the hearsay rule apply. Therefore, the statements are inadmissible. Answer choice B is incorrect. The fact that the woman is available to testify would not otherwise preclude the admission of the technician's testimony concerning the woman's statements. However, the testimony is not admissible because it does not fall under a hearsay exception. Answer choice C is incorrect. Although the exposure to legal liability would be sufficient for the woman's statements to fall within the statement against interest exception to hearsay, this exception only applies if the declarant is unavailable to testify. Here, the facts indicate the woman is available to testify. Answer choice D is incorrect. Although a statement made for medical diagnosis or treatment need not be made to a physician, the statement must be made for medical diagnosis or treatment. Even if the woman's statement that the climbers had ingested hallucinogenic drugs before the climb was medically relevant to her treatment or diagnosis, her claim that the man unexpectedly jumped does not fall under this exception.

An author filed a complaint in federal district court as required by federal law alleging copyright infringement by a corporate publisher. The corporation was incorporated and has its headquarters in the state in which the federal district court is located. Sixty-five days after filing the complaint, the author served the complaint and summons on the secretary for the president of the corporation while she was at her desk in the company headquarters. State law permits process to be served on a corporation by leaving the summons and complaint at the defendant's principal place of business with a person of suitable age and discretion who is employed there. Which of the following would likely serve as the best grounds upon which to challenge the service of process as improper? A. The author served the process. B. Federal rules generally do not permit service of process on an employee of a corporation unless the employee is an officer of the corporation or an agent appointed or authorized to receive process. C. State rules for service of process do not apply when the cause of action is not based on diversity jurisdiction. D. The service of process was untimely.

Answer Choice A. Service of the complaint and summons may be made by any nonparty who is at least 18 years old. Consequently, service of process by the author, who is the plaintiff in this action, was improper. Answer choice B is incorrect because, while FRCP 4 generally requires that service on a corporation in the United States be effected by delivering the summons and complaint to an officer, managing agent, general agent, or agent appointed or authorized by law to receive process, this rule also permits that service of process may be made by following state law for serving a summons in an action brought in courts of general jurisdiction in the state where the district court is located or where service is made. Answer choice C is incorrect because the federal rule that also permit that service of process may be made by following state law for serving a summons in an action brought in state court, is not restricted to diversity actions. Answer choice D is incorrect. Unless service is made in a foreign country, the summons and complaint generally must be served within 90 days after filing the complaint. Here, since the individual effected service 65 days after filing the complaint, service of process was timely.

After receiving a grant of use and derivative use immunity, a mayor testified before a grand jury regarding allegations of corruption. Subsequently, the mayor was indicted for the crime of extortion by a public official based on evidence the prosecutor obtained without the use of the mayor's grand jury testimony. At trial, the mayor testified on his own behalf. Is it constitutionally proper for the prosecutor to impeach the mayor's testimony with a statement that the mayor made before the grand jury? A. No, because the mayor's grand jury testimony was coerced by the grant of immunity. B. No, because the mayor did not waive his Fifth Amendment Privilege Against Self-Incrimination by taking the stand. C. Yes, because the prosecutor indicted the mayor without the use of the mayor's grand jury testimony. D. Yes, because statements otherwise protected by the Fifth Amendment may be used for impeachment purposes.

Answer Choice A. Testimony given under a grant of immunity is considered coerced and involuntary. A defendant's involuntary statement, such as a confession produced by coercion, cannot be used either substantively or for impeachment purposes. Consequently, the mayor's statement made under the compulsion of the grant of immunity may not be used to impeach the mayor's testimony at his trial for an offense that was the subject of the mayor's grand jury testimony. Answer choice B is incorrect because generally a defendant does waive his Fifth Amendment Privilege Against Self-Incrimination by taking the stand; having taken the stand, the defendant cannot assert the privilege in response to the prosecution's proper cross-examination of his testimony, including impeachment questions. However, in this case, the prosecution is prohibited from using the mayor's involuntary statement because to do so would constitute a denial of due process. Answer choice C is incorrect. Because the mayor only had use and derivative use immunity, prosecuting him based on evidence obtained without the use of the his grand jury testimony does not violate the mayor's Fifth Amendment Privilege Against Self-Incrimination. However, the prosecutor's use of a statement made by the mayor under the compulsion of the grant of immunity in trial to convict the mayor of a crime about which the mayor testified does constitute a denial of due process. Answer choice D is incorrect because, while a defendant's statement taken in violation of Miranda may be used to impeach the defendant's testimony at trial, there is no similar exception for use of a statement made under a grant of immunity because such statement is viewed as having been coerced.

Congress enacted a statute that required a state "to take appropriate action to overcome language barriers that impede equal participation by its students in its instructional programs." The statute was enacted in response to Congress's concern that public school students whose native language was not English were hampered in their ability to learn because classroom instruction was almost exclusively presented in English, and that a state's failure to take action to address this issue resulted in the denial of equal educational opportunities to students on the basis of their race or national origin. Among the approved appropriate state actions is the funding of special classes for such students. Of the following, which constitutes the best source of constitutional authority for this statute? A. Section 5 of the Fourteenth Amendment. B. The General Welfare Clause of Article I, Section 8. C. The Equal Protection Clause of the Fifth Amendment. D. The Spending Clause of Article I, Section 8.

Answer Choice A. The Section 5 Enabling Clause of the Fourteenth Amendment permits Congress to pass legislation to enforce the equal protection and due process rights guaranteed by the Fourteenth Amendment, as long as there is "congruence and proportionality" between the injury to be prevented or remedied and the means adopted to achieve that end. Congress may override state government action that infringes upon Fourteenth Amendment rights, but it may not regulate wholly private conduct under this amendment. Answer choice B is incorrect because the General Welfare Clause has been interpreted as giving Congress the power to tax and spend for the general welfare, but not as permitting Congress to enact legislation simply for the general welfare, as a state legislature can under its police power. The General Welfare Clause is inapplicable here because the federal statute at issue authorizes the expenditure of state funds to fund these classes, not federal funds. Answer choice C is incorrect. There is no federal equal protection clause. Although there is no federal equal protection clause, the Supreme Court has held that the Fifth Amendment Due Process Clause includes the rights guaranteed by the Equal Protection Clause of the Fourteenth Amendment. However, this only means that discrimination by the federal government is subject to the same standard as discrimination by the states—it does not give Congress the power to pass legislation to enforce the equal protection and due process rights guaranteed by the Fourteenth Amendment. Answer choice D is incorrect. Although Congress has the power pursuant to the Spending Clause to authorize the expenditure of federal funds for the general welfare, this statute does not authorize such an expenditure. Instead, the statute sanctions a state's allocation of its funds in a particular manner to meet the objectives of this statute.

A man and woman who were co-workers were brought into the station house by police for questioning regarding embezzlement from their employer. Each voluntarily made a statement to police after being properly Mirandized. They were jointly tried before a jury, and both elected to testify at trial. The woman's statement to police at the station house implicated both the man and the woman in the embezzlement. The man's objection to the admission of the woman's statement at trial, which implicated him, was overruled. When the woman took the stand, she repeatedly denied making a self-incriminating statement at the police station, even under cross-examination by the man's attorney. Both were convicted of embezzlement. The man has appealed his conviction, seeking a reversal on the basis of the admission of the woman's statement at the trial. How should the appellate court rule? A. Uphold the man's conviction, because the woman testified at the trial. B. Uphold the man's conviction, because the man testified at the trial. C. Reverse the man's conviction, because the woman denied making the statement. D. Reverse the man's conviction, because admission of the woman's hearsay statement violated the Confrontation Clause.

Answer Choice A. The admission against a defendant of a confession that implicates the defendant made by a non-testifying co-defendant at a joint trial violates the defendant's Sixth Amendment right to confrontation. However, when the co-defendant testifies, the defendant cannot complain that this right has been denied because he could confront the witness on cross-examination. In this case, the co-defendant, the woman, testified at the trial. Accordingly, the man's Sixth Amendment rights were not violated. Answer choice B is incorrect because whether a defendant testifies at trial is irrelevant to the issue of whether the admission of a co-defendant's out-of-court statement violates the defendant's Sixth Amendment right to confrontation. Answer choice C is incorrect. Although the woman denied making the statement, she took the witness stand and was subject to cross-examination by the man's attorney. Therefore, the man's right to confront the witness has not been violated. Answer choice D is incorrect. Although the defendant has a Sixth Amendment right to confrontation that generally prevents the introduction of a hearsay statement against the defendant, the Confrontation Clause places no constraints on the use of the declarant's prior out-of-court testimonial statements when the declarant is available for cross-examination at trial.

During wartime, a number of military aircraft factories expanded their operations and hired many additional employees. As a result of this increased employment, these communities experienced a sudden and unprecedented need for affordable rental housing. In response to this need, Congress passed a law establishing the maximum rent that may be imposed for a two-bedroom apartment within a ten-mile radius of a military aircraft factory. Is this statute likely to be constitutional? A. Yes, pursuant to the congressional war and defense powers provided by Article I, Section 8. B. Yes, because the statute is rationally related to a legitimate governmental interest. C. No, because the statute impermissibly regulates private conduct. D. No, because Congress is not authorized to legislate for the general welfare.

Answer Choice A. The authority granted to Congress under the war power of Article I, Section 8, is very broad. Congress may take whatever action it deems necessary to provide for the national defense in both wartime and peacetime, including rent control of the civilian economy during wartime, and even during the post-war period. Therefore, this statute is constitutional pursuant to the congressional war and defense powers. Answer choice B is incorrect because rational basis analysis does not apply to legislation passed pursuant to congressional war and defense powers. Answer choice C is incorrect because rent control is not an impermissible limitation on private conduct when enacted under the congressional war and defense powers. Answer choice D is incorrect. Although the General Welfare Clause gives Congress broad power in exercising its spending and taxing powers, it does not give Congress the specific power to legislate for the public welfare in general. However, Congress may rely on other powers to pass legislation that supports the general welfare as long as the legislation also falls under that power.

Congress has created a for-profit corporation to provide passenger railroad service in the United States. The governing statute provides that the corporation is not an agency or instrumentality of the federal government. Pursuant to the statute, six out of the eight members of the corporation's board of directors are appointed by the President. Of those six members, four are subject to the advice and consent of the Senate. A private individual sought to advertise a political message on an electronic billboard inside a train station owned by the corporation. The corporation had an advertising policy, adopted by the board of directors, of rejecting all political advertisements. Based on this policy, the corporation refused to allow the advertisement. The individual has challenged this refusal as a violation of his First Amendment free speech rights. Does the corporation's refusal constitute state action for purposes of the individual's constitutional challenge? A. Yes, because the corporation is created and controlled by the federal government. B. Yes, because the denied message was political in nature. C. No, because the governing statute specifically provides that the corporation is not an agency or instrumentality of the federal government. D. No, because the corporation is a for-profit entity.

Answer Choice A. The corporation was created and controlled by the federal government. As such, an action taken by the corporation in accord with a policy adopted by the corporation's board of directors constitutes governmental action for purposes of determining whether the action violates the plaintiff's constitutional rights under the First Amendment. Answer choice B is incorrect because, while the nature of the message that the individual sought to advertise can affect whether the corporation's rejection of the advertisement is constitutionally permissible, the message and its nature is not relevant to whether the corporation is a governmental entity. Answer choice C is incorrect. While congressional characterization of an entity as a non-governmental agency may be controlling with regard to issues such as applicability of the doctrine of sovereign immunity to a tort action brought against the entity, a corporation created and controlled by the federal government is part of the government for the purposes of the First Amendment. Answer choice D is incorrect because, although the corporation is a for-profit entity, the creation and control of the corporation by the federal government is determinative of whether its conduct constitutes governmental action for the purposes of this constitutional challenge to its action.

A chicken farmer raised chickens to sell to people who wanted to eat fresh, organic, cage-free eggs at a lower cost than buying from a store. A nutritionist who was interested in raising chickens contacted the chicken farmer. The two entered into a written agreement that contained a clause stating that the agreement was the final and complete agreement between them. Prior to finalizing the contract, the farmer and the nutritionist had spoken on the phone, and they orally agreed that the farmer would not sell the chickens to the nutritionist unless she built a chicken coop for the chickens. After the contract was signed, the nutritionist attempted to have a chicken coop built, but due to the difficulty of building one in her small backyard, the coop would not be completed until after the chickens were delivered. On the day of delivery, the farmer refused to sell the chickens to the nutritionist when he saw that the chicken coop was not habitable. Is the farmer in breach of contract? A. No, because building the chicken coop was a condition precedent. B. No, because the agreement to build the chicken coop was oral, not written. C. Yes, because the nutritionist made a good-faith effort to build the coop. D. Yes, because the oral agreement occurred before the contract was finalized.

Answer Choice A. The parol evidence rule generally prevents a party to a written contract from presenting prior extrinsic evidence that contradicts the terms of the contract as written. However, parol evidence may be admitted to prove a condition precedent to the existence of the contract. Here, the farmer agreed to sell the chickens to the nutritionist on the condition that she build a chicken coop to house the chickens. Although the oral agreement regarding the chicken coop occurred prior to the finalization of the contract, it is admissible as a condition precedent. Thus, the farmer is not in breach of contract because the nutritionist did not satisfy the condition precedent to the existence of the contract for the chickens. For this reason, answer choice D is incorrect. Answer choice B is incorrect because a condition precedent does not have to be written; it can also be oral. Answer choice C is incorrect. Despite the nutritionist's good-faith effort to build the coop, she has not satisfied the condition precedent. Consequently, the farmer is not in breach of contract.

A movie director entered into a two-year rental agreement with the CEO of a production company to rent one of the company's 20 studios for $3,000 a month. The agreement also included an option that stated in its entirety, "At his option, [the director] may purchase one of the company's studios prior to the end of the rental agreement." Prior to signing the agreement, the CEO made a promise to add new editing equipment to the rented studio, but did not follow through on the promise. Before the rental agreement expired, the director refurbished the studio he was renting in order to enhance its editing quality. The director then attempted to pay $100,000 in order to exercise his option and purchase the studio. The CEO refused to sell the studio to the director. What is the strongest argument in favor of the CEO if the director sues him for specific performance? A. Terms of the option to purchase a studio are too indefinite. B. The CEO's oral promise to add editing equipment was a condition precedent to his duty to perform. C. The option to purchase the studio by the director violates the parol evidence rule. D. There is no consideration to support the option to purchase a studio at the production company.

Answer Choice A. Under common law, all essential terms (i.e., the parties, subject matter, price, and quantity) must be covered in an agreement. Here, the rental agreement did not describe the essential terms of the option to purchase, including the cost of the studio or the specific studio that was for sale. Answer choice B is incorrect because the CEO would not be able to benefit from his own failure to perform. Answer choice C is incorrect. The parol evidence rule prevents parties from altering their written contract through the use of contemporaneous or prior oral declarations. Here, the option to purchase was included in the written rental agreement. Therefore, it does not violate the parol evidence rule. Answer choice D is incorrect. If an option is within an existing contract, no separate consideration is required for the option to be enforceable. Here, the option was included in the rental agreement. The rental agreement included consideration in the form of the director's monthly payment of rent to the company. Therefore, no separate consideration for the option is required.

The U.S. Army Corps of Engineers ("Corps") managed a reservoir and allocated water from the reservoir between two states. One of the states, dissatisfied with the allocation, sued the Corps in a federal district court. The other state, supporting the current allocation, intervened as a defendant and filed a motion to dismiss for lack of jurisdiction, claiming that the United States Supreme Court should hear the matter. The federal district court properly found that the plaintiff-state was seeking relief from the Corps, not from the defendant-state. Based on this finding, the court held that the action did not involve a controversy between two states, and that the federal district court therefore retained original jurisdiction over the action. The defendant-state has challenged this ruling. Is the federal district court constitutionally required to dismiss this action because original jurisdiction lies with the Supreme Court? A. No, because Congress may and has denied the Supreme Court exclusive original jurisdiction over a controversy involving only one state. B. No, because the Constitution only requires that the Supreme Court have appellate jurisdiction over a controversy involving a state. C. Yes, because the underlying controversy involves two states, even if the second state is not officially a party to the action. D. Yes, because the action involves a controversy between a state and an entity of the United States.

Answer Choice A. While the U.S. Constitution gives the U.S. Supreme Court original jurisdiction over, among other matters, a controversy involving a state as a party, Congress may grant concurrent original jurisdiction to lower federal courts. By statute, Congress has granted federal district courts concurrent original jurisdiction over all controversies over which the Constitution granted the U.S. Supreme Court original jurisdiction, except for a controversy between two states. Since the district court properly determined that the current action was not a controversy between two states, the district court is not constitutionally required to dismiss this action. Answer choice B is incorrect because, although the U.S. Supreme Court primarily exercises appellate jurisdiction, the U.S. Constitution provides that the Supreme Court has original jurisdiction over "all cases affecting ambassadors, other public ministers and consuls and those in which a State shall be a party." Therefore, this answer choice is a misstatement of the law. Answer choices C and D is incorrect. The U.S. Constitution does provide that the U.S. Supreme Court has original jurisdiction over, among other matters, a controversy involving a state as a party. However, the U.S. Constitution does not grant the U.S. Supreme Court exclusive original jurisdiction over an action involving a state, an entity of United States, or action involving a controversy between a state and an entity of the United States.

In which of the following situations is the defendant most likely to be guilty of the crime charged? (A) without the permission of the owner, the defendant takes a car with the intention of driving it three miles to a grocery store and back. The defendant is charged with larceny. (B) the defendant gets permission to borrow a car for the evening by falsely promising to return it, although he does not intend to do so. Two days later, he changes his mind and returns the car to it owner. The defendant is charged with larceny by trick. (C) The defendant gets permission to borrow a car for the evening by misrepresenting his identity and falsely claiming he has a valid driver's license. He returns the car the next day to the car's owner. The defendant is charged with obtaining property by false pretenses. (D) With permission, the defendant, promising to return it by 9:00 pm, borrows a car. Later in the evening, the defendant decides to keep the car until the next morning and does so. The defendant is charged with embezzlement.

Answer Choice B is the best choice. Because the defendant satisfies all the elements of larceny by trick. The question asks the choice where the defendant is most likely to be convicted. This means in one choice the defendant will be guilty, and in the other three he won't. So you have to determine the elements in each choice, and apply them mechanically, to arrive at the best response. Larceny by trick is differnet from larceny in the way that the property is acquired; larceny by trick involves a fraudulent taking, which negates the owner's apparent consent. Every other element is th3e same: the carrying away of another's personal property, with the intent to steal it. Under the facts here, these elements are satisfied: the defendant obtains possession by misrepresenting his intent to return the car. The wrinkle here is that, after taking the car, the defendant decides to return it after all. However, the defendant's subsequent actions are not relevant to his guilt- once his has fulfilled all the requirements of larceny by trick, his subsequent repentance cannot exonerate him. Thus, the defendant is likely to be guilty, making (B) the best response.

A man suspected a neighbor had slashed his tires. The man wrote the neighbor a letter accusing the neighbor of slashing his tires because the man refused to remove a large pink flamingo decoration from his front yard after repeated requests from the neighbor. The man put the letter in the neighbor's mailbox. The next night, the man received a handwritten note under his door. The note admitted to slashing the tires and confirmed that the act was in retaliation for the "atrocious florescent eyesore" in the man's front yard. The note was signed with the neighbor's name. The man brought a tort action against the neighbor and intends to introduce the note received by the man into evidence at trial. The neighbor objected to the introduction of the handwritten note, claiming that it cannot be properly authenticated. How should the court rule? A. Admit the note as a self-authenticating document, because it was signed by the neighbor. B. Admit the note, because it is authenticated under the reply letter doctrine. C. Exclude the note, unless a handwriting expert can verify that the neighbor wrote the note. D. Exclude the note, because it was not signed under penalty of perjury.

Answer Choice B. A document may be authenticated by evidence that it was written in response to a communication, so long as it is unlikely, based on the contents, that it was written by someone other than the recipient of the first communication. In this case, the man wrote to his neighbor about his slashed tires and the decoration in his yard, and the note in response confessed to the accusation in the letter and contained specific details about the incident, as well as the neighbor's name. These facts indicate that the neighbor, the recipient of the man's letter, wrote the response. Answer choice A is incorrect because the signature does not make this reply a self-authenticating document. Answer choice C is incorrect because in this situation, a handwriting expert is not necessary to verify that the letter was written by the neighbor, as the reply letter doctrine applies to authenticate the note. Answer choice D is incorrect because it is not necessary for the note to be signed under the penalty of perjury to be authenticated.

A driver was speeding through a town's business district. The driver swerved suddenly to avoid hitting a car that was double parked in the middle of a traffic lane, and hit a pedestrian who was crossing the street. The pedestrian filed a complaint in federal district court in the state where the accident occurred against the owner of the car that was double parked. The pedestrian argued that the owner's negligence in leaving his car in the middle of a traffic lane caused the pedestrian to sustain physical injuries, and asserted $100,000 in damages. The plaintiff-pedestrian is a citizen of the state where the accident occurred, and the defendant is a citizen of a neighboring state. The driver of the speeding car is a citizen of the state where the accident occurred and is in the midst of a personal bankruptcy action. The defendant filed a motion to dismiss, arguing that the driver of the speeding car was an indispensable party. Applicable state law provides that tortfeasors may be jointly and severally liable for a plaintiff's injuries. Is the court likely to grant the defendant's motion to dismiss? A. No, because the driver is judgment-proof due to his bankruptcy. B. No, because the driver is not subject to compulsory joinder. C. Yes, because joining the driver would defeat the court's jurisdiction. D. Yes, because the driver's participation is necessary for a just adjudication.

Answer Choice B. A person who is subject to service of process and whose joinder will not deprive the court of subject-matter jurisdiction or destroy venue must be joined as a party if the party's participation in the lawsuit is necessary for a just adjudication. Tortfeasors facing joint and several liability are not considered necessary parties. In this case, the driver was a joint tortfeasor and thus was not a necessary party, so the court was correct to allow the case to proceed without the driver. Answer choice A is incorrect because a joint tortfeasor is not a necessary party regardless of whether the individual is judgment-proof. Answer choice C is incorrect because the driver is not a necessary party, and thus the issue of jurisdiction is not relevant. Under certain circumstances, an action may be dismissed if compulsory joinder cannot occur because it would destroy diversity. Because the driver is not a necessary party, however, that analysis is not necessary in this case. Answer choice D is incorrect because a joint tortfeasor is not a necessary party.

A husband and wife decided to take a luxury cruise. On the last night of the cruise, the husband passed away in his sleep. The next morning, a temporary coffin was brought onto the ship to transport the husband's body to a mortuary. The cruise line used a crane to transport the coffin from the upper deck of the ship to the dock. The crew members negligently attached the crane straps to the coffin, and as the coffin was transported over the water, the straps loosened, and the husband's body fell out of the coffin and into the harbor. The wife, who watched her husband's body fall into the water, was horrified and traumatized by the incident. The wife filed suit against the cruise line for damages resulting from her emotional distress. Will the wife prevail? A. Yes, because the wife watched her husband's body fall into the harbor. B. Correct Answer: Yes, because the crew members did not properly secure the coffin. C. No, because the wife did not suffer any physical injury from the accident. D. No, because the crew members' actions were not extreme and outrageous.

Answer Choice B. A plaintiff can recover for negligent infliction of emotional distress from a defendant whose tortious conduct placed the plaintiff in harm's way if the plaintiff demonstrates that: (i) he was within the "zone of danger" of the threatened physical impact—that he feared for his own safety because of the defendant's negligence; and (ii) the threat of physical impact caused emotional distress. Under the majority rule, damages for negligent infliction of emotional distress without physical symptoms are not recoverable; however, an exception exists for the negligent mishandling of a corpse. Here, the crew members mishandled the husband's corpse when they failed to properly secure the crane straps to the coffin. Due to their negligence, the wife will prevail. Answer choice A is incorrect because the simple fact that the wife saw her husband's body fall in the water does not give her a successful claim. If the body had fallen due to no fault on the part of the cruise line, she could not recover. It was the crew members' negligent handling of the body that is determinative of her claim, making answer choice B the better answer. Answer choice C is incorrect because, as discussed above, the negligent mishandling of a corpse is an exception to the physical injury requirement of negligent infliction of emotional distress claims. Answer choice D is incorrect because it applies the requirement for the nature of the defendant's conduct associated with intentional, not negligent, infliction of emotional distress. Even though the crew members' conduct was not extreme and outrageous, this does not bar the woman's recovery under negligent infliction of emotional distress.

A grand jury indicted the defendant on drug trafficking charges. A week after his indictment and subsequent arrest, the defendant was brought before a judge to determine bail. The judge denied bail, and refused to hear arguments from the defendant's counsel that there was not probable cause for the defendant's detention. Following the bail hearing, the defendant's counsel filed a motion to dismiss the charges against the defendant, arguing that the defendant was denied his right to a preliminary hearing as to whether there was probable cause for the defendant's detention. The judge promptly denied the defendant's motion, and scheduled the case for trial. Did the judge err in denying the defendant's motion? A. No, because the defendant waived his right to a preliminary hearing by failing to request it within 48 hours of his arrest. B. No, because there was no need for a determination of probable cause by a judge. C. Yes, because a defendant is entitled to a determination of probable cause by a judge. D. Yes, because the defendant did not waive his right to a preliminary hearing. Rationale

Answer Choice B. A preliminary hearing to determine whether probable cause exists to hold the defendant (i.e., a Gerstein hearing) generally must be held within 48 hours of the defendant's arrest. There is no need to hold a preliminary hearing if probable cause has already been determined through a grand jury indictment or an arrest warrant. Answer choice A is incorrect because the defendant was not entitled to a preliminary hearing, and thus there is no issue of waiver. Answer choice C is incorrect because a defendant is not entitled a determination of probable cause to detain following arrest when the defendant was arrested following a grand jury indictment or pursuant to an arrest warrant. Answer choice D is incorrect because the defendant was not entitled to a preliminary hearing under the circumstances, and thus there is no issue of waiver.

A chef decided to start a food truck business. He borrowed $60,000 from a bank to buy a food truck, and secured the loan by mortgaging his home. The bank recorded the mortgage the day after this loan was made. The following week, the chef borrowed an additional $50,000 from his mother to buy cooking appliances for the truck. He also secured the loan by a mortgage on his home, and his mother recorded this mortgage in a timely manner. The food truck business proved to be a bad investment for the chef, and he was unable to make any payments on his bank loan. He approached his father for help paying off the bank loan and his father agreed to loan him $40,000, secured by a mortgage on the chef's home. The father recorded his mortgage in a timely manner. The father believed that the bank loan would be paid off in its entirety by the father's $40,000 loan. He was unaware of the loan by the chef's mother, who was his ex-wife. The chef used the $40,000 from his father's loan to partially pay off the bank loan. When the chef failed to make monthly payments to his mother, she brought a foreclosure action against the chef. In the foreclosure action, whose mortgage will be deemed to have priority? A. The mother's mortgage, as a purchase-money mortgage. B. The mother's mortgage, because the father recorded his mortgage after the mother did. C. The father's mortgage, because his loan was used to pay off the bank's mortgage. D. The father's mortgage, because otherwise the chef would be unjustly enriched.

Answer Choice B. All jurisdictions have enacted recording statutes. Regardless of which recording act applies, the mother's mortgage has priority because she recorded first. Answer choice A is incorrect because while a purchase-money mortgage can have priority over various other interests, in this case, neither the father nor the mother had a purchase-money mortgage; the mother's loan was used to purchase equipment, and her mortgage was on her son's home. The mother's mortgage has priority due to her timely recording. Answer choice C is incorrect. A person who pays off another person's mortgage obligation becomes the owner of the obligation and the mortgage to the extent necessary to prevent unjust enrichment. This is known as subrogation. Here, the father paid off $40,000 of the chef's mortgage obligation to the bank, but subrogation is not permitted when the full obligation secured by the mortgage is not discharged. Answer choice D is incorrect because, although the chef may be unjustly enriched, a partial discharge is not enough to subrogate the bank's mortgage and give the father's mortgage priority over the mother's mortgage.

A loan shark instructed a henchman to either collect a debt from a borrower or shatter his kneecaps as a message to other borrowers of the importance of timely repayment of their loans. The henchman, mistaking the borrower's twin brother for the borrower, shattered the twin's kneecap when the twin refused to pay back a debt he had never incurred. The twin died from the shattered kneecap when a fragment of bone severed an artery and he bled to death. Is the loan shark likely to be found guilty of attempted murder of the borrower? A. No, because a mistake of fact is not a defense to a malice crime such as murder. B. No, because the loan shark did not intend for the shattering of the borrower's kneecaps to result in his death. C. Yes, because the loan shark is vicariously liable for the acts of his agent, the henchman. D. Yes, under the doctrine of transferred intent.

Answer Choice B. Although murder is a malice crime that can be satisfied by an intent to do serious bodily harm, attempted murder is a specific-intent crime. Here, the loan shark had the intent to do serious bodily harm to the borrower by subjecting him to a shattered kneecap. However, the loan shark did not possess the intent to kill the borrower. Therefore, even though the loan shark took a substantial step towards the murder of the borrower in directing his henchman to shatter the borrower's kneecaps, the loan shark is likely not guilty of attempted murder because attempt requires specific intent. Answer choice A is incorrect. A mistake of fact can be a defense to a malice crime such as murder if the mistake is a reasonable one. A mistake of fact can also be a defense to a specific-intent crime such as attempted murder even if the mistake of fact is an unreasonable one. Mistaking the borrower's twin for the borrower is likely a reasonable mistake. More importantly, the mistake did not result in the death of the borrower or even an attempt being made on his life, but instead in the death of the borrower's twin brother. Answer choice C is incorrect. Generally, vicarious-liability crimes are limited to regulatory crimes, and punishment is limited to fines. Instead, if the loan shark is liable for the actions of his henchman, it will be because the loan shark is an accomplice who is also responsible for the crime committed at his direction by his henchman. Answer choice D is incorrect because the doctrine of transferred intent can make the loan shark and his henchman liable for the murder of the borrower's twin brother, but it is inapplicable with respect to the borrower himself. Further, attempt is a specific-intent crime.

A defendant has been charged with battery. While waiting to bat in a game between two amateur teams in a recreational softball league, the defendant was taking practice swings. Unlike the other players in the league, who took care to look behind them before taking a practice swing, the defendant undertook no similar precaution. After stopping for about a minute, the defendant resumed his practice swings. On the follow through of his first practice swing after restarting, the defendant carelessly struck a team member in the head, causing serious injury. The team member, who was facing away from the defendant at the time, had inadvertently moved into the path of the defendant's bat, unaware that the defendant had resumed his practice swings. Is the defendant likely guilty of battery? A. Yes, even though the victim had impliedly consented to the harmful contact by participating in the softball game. B. Yes, because the defendant's failure to take precautions was a gross deviation from the standard of care of a reasonable person in the same situation. C. No, because the victim was unaware that the defendant had resumed his practice swings and that he was about to be hit. D. No, because the defendant lacked the specific intent to harm the victim.

Answer Choice B. Battery is the unlawful application of force to another person that causes bodily harm or constitutes an offensive touching. Battery is a general-intent crime that includes not only intentional conduct but also criminal negligence. Here, the defendant's practice swing caused a serious head injury to a team member. His failure to take a precaution undertaken by the other members in the softball league suggests that this failure was not mere negligence, but instead was a gross deviation from the standard of care of a reasonable person in the same situation, thus constituting criminal negligence. Answer choice A is incorrect. Although the team member is deemed to have consented to harmful contact that results from participating in the softball game, such as being tagged out by an opposing player or being hit by a softball on the field, it is unlikely that such consent extends to conduct that does not occur as a direct consequence of playing the game itself. Answer choice C is incorrect because battery does not require that the victim be apprehensive of an imminent bodily harm or offensive contact. Answer choice D is incorrect. Although a defendant who acts with the specific intent to cause bodily harm or an offensive touching can be guilty of battery, specific intent is not required. Battery is a general-intent crime that includes not only intentional conduct but also criminal negligence.

A well-known celebrity gossip blogger made a post on his website that was highly critical of a famous celebrity. The blogger called the celebrity a "drug addict" in need of serious help. Upon learning of the post, the celebrity filed an action for defamation against the blogger. At trial, the blogger called a witness who testified that she had used cocaine with the celebrity on at least two occasions within the past two months. The celebrity objected to the witness's testimony. Is the witness's testimony admissible? A. Yes, because the exclusion of character evidence only applies to a criminal trial. B. Yes, because the celebrity's character is an essential element in this trial. C. No, because the celebrity's character may only be proved by reputation or opinion testimony. D. No, because the witness, a drug user, is not credible.

Answer Choice B. Character evidence is admissible when character is an essential element of a claim or defense, rather than a means of proving a person's conduct. Character is most commonly an essential element in defamation cases. When character evidence is admissible as evidence in a civil case, it may be proved by specific instances of a person's conduct as well as either by testimony about the person's reputation or by testimony in the form of an opinion. Here, the celebrity brought an action for defamation against the blogger. Accordingly, the celebrity's character is an essential element and it may be proven by specific instances of the celebrity's conduct. Therefore, the witness's testimony regarding the celebrity's use of cocaine on a number of occasions is admissible. Answer choice A is incorrect. While there are significant restrictions on the use of character evidence in criminal cases to show that a defendant acted in conformity with a character trait, the use of evidence of a character trait is also generally prohibited in civil cases as well. Answer choice C is incorrect because the celebrity's character may also be proved by specific instances of conduct. Answer choice D is incorrect. The credibility of the witness is determined by the jury and is not a valid reason for the court to prohibit the admission of the witness's testimony.

Congress created an investigatory committee to look into the judicial administrative processes of federal judges. In particular, the committee was tasked with investigating the issuance of court orders, the management of trial schedules, and the drafting of basic court documents by judicial assistants. The goal of the committee was to ensure that the administrative processes were carried out efficiently, accurately, and pursuant to federal legislation regarding the judicial administrative process. After much investigation, the committee concluded that a federal district court judge was negligently mismanaging his trial schedule, causing undue delays in his case log. Based upon its findings, the committee made a recommendation to Congress to remove the federal judge. Can Congress remove the federal judge based on the committee's recommendation? A. No, because Congress cannot investigate a federal judge's administrative processes. B. No, because the federal judge may only be removed by impeachment. C. Yes, because Congress can remove federal judges with or without cause. D. Yes, because the federal judge mismanaged his trial schedule.

Answer Choice B. Congress is limited in its restraints on the independence of the judiciary, and in particular under Article III, Section 1, as to its ability to remove or impeach a federal judge. Federal judges may "hold their offices during good behavior" and may be removed only by impeachment. Here, impeachment proceedings were not brought against the federal judge. Rather, the removal was sought based on the investigatory committee's recommendation. Moreover, although the federal judge in this case mismanaged his trial schedule, this does not violate the good behavior clause of the Constitution. A violation of the good behavior clause would more likely arise in a scenario where a judge has accepted a bribe or committed perjury. Answer choice A is incorrect because Congress can investigate judicial administrative processes. Answer choice C is incorrect because Congress cannot remove federal judges without proper impeachment proceedings based on a lack of "good behavior." Answer choice D is incorrect because, as stated above, mismanagement of a trial schedule likely does not violate the good behavior clause.

A collector of historical documents discovered that a letter she had purchased many years earlier that she thought was in the handwriting of a famous historical figure was a forgery. Because the person who sold her the letter was now dead, the collector advertised the letter for sale as genuine and priced it accordingly. The benefactor of a museum expressed interest in purchasing the letter. Before doing so, the benefactor had a curator at the museum examine the letter. The curator mistakenly informed the benefactor that the letter was genuine. As a consequence, the benefactor purchased the letter, paying the collector in cash. Can the collector be charged with false pretenses? A. No, because the collector did not forge the letter. B. No, because the benefactor did not rely on the collector's representation of the letter as genuine. C. Yes, because the collector advertised and priced the letter as genuine. D. Yes, because the benefactor paid the collector for the letter.

Answer Choice B. False pretenses requires (i) obtaining title to the property (ii) of another person (iii) through the reliance of that person (iv) on a known false representation of a material past or present fact, and (v) the representation is made with the intent to defraud. Here, although the collector advertised the letter as genuine and obtained money from the benefactor for the letter, the benefactor did not purchase the letter in reliance on the collector's representation, but instead on the museum curator's assessment of the letter's authenticity. Consequently, the collector cannot properly be charged with false pretenses. Answer choice A is incorrect. Although the collector did not forge the letter, the collector did misrepresent its authenticity. Consequently, had the benefactor purchased the letter in reliance on the collector's misrepresentation, the collector could have been charged with false pretenses even though the collector did not forge the letter. Answer choice C is incorrect because, although the collector misrepresented the authenticity of the letter, the benefactor did not rely on this misrepresentation in purchasing the letter. Answer choice D is incorrect. Although the collector did receive money from the benefactor for the letter and thus obtained title to the property of another, the benefactor not did rely on the collector's misrepresentation, and therefore the collector cannot properly be charged with false pretenses.

A wife brought a divorce action against her husband in State A, where they were both domiciled. As part of the divorce decree, the court ordered the husband to transfer to his wife title to land held in his name only. The land was located in State B. After the husband conveyed the land to his wife, the husband deeded the land to a friend in exchange for a payment equal to the fair market value of the land. The friend was domiciled in State B and aware of the husband's prior conveyance of the land. The friend recorded his deed in State B, before the wife recorded her deed. State A has a race-notice recording statute, while State B has a race recording statute. In an action to quiet title brought by the friend against the wife in a State B court, who is likely to prevail? A. The friend, because he was domiciled in State B. B. The friend, because the law of State B applies in determining ownership of the land. C. The wife, because her ownership of the land arises from the divorce decree of a State A court. D. The wife, because the friend had knowledge of the husband's conveyance of the land to her.

Answer Choice B. Generally, real property is subject to the law of the situs. Here, the recording act of State B is a race-type law. Consequently, because the friend, who purchased the property, recorded his deed first, the friend prevails over the wife. Answer choice A is incorrect because, in determining who holds title to a real property interest, the law of the state in which the property is located generally applies when there is a conflict between that law and the law of another state. This is particularly true when the applicable recording act is outcome determinative. The domicile of one of the parties contesting ownership is not a significant factor in determining the applicable law in such cases. Answer choice C is incorrect because, while the wife's ownership of the land does stem from a divorce decree by a State A court, this decree concerns ownership of the land as between the wife and her husband. The case at hand deals with the ownership of the land between the wife and the friend. Answer choice D is incorrect. Although the friend's knowledge of the husband's prior conveyance of the land to the wife would prevent him from asserting priority over the wife's interest under the race-notice recording act of State A, there is a conflict-of-laws issue between the recording acts of State A and State B. Thus, the recording act of State B governs because the land is located in State B.

A patient brought a suit against a physician in federal district court under diversity jurisdiction, alleging medical malpractice based on state law. After the patient presented his case to the jury, the physician moved for judgment as a matter of law. In deciding the motion, the court reviewed all of the evidence presented by the patient in the light most favorable to the patient. The court considered the patient's only two witnesses to not be credible and gave no weight to their testimony. As a result, the court determined that the patient did not present sufficient evidence for a jury to reasonably return a verdict in the plaintiff's favor. Accordingly, the court granted the physician's motion for judgment as a matter of law. Was the court's ruling proper? A. No, because the motion cannot be made until both parties have been fully heard. B. No, because the court considered the credibility of the patient's witnesses and the weight of the evidence. C. Yes, because the court viewed the evidence in the light most favorable to the patient. D. Yes, because the court determined that patient's witnesses were not credible.

Answer Choice B. In considering a motion for judgment as a matter of law, the court must view the evidence in the light most favorable to the opposing party and draw all reasonable inferences from the evidence in favor of the opposing party. It may not consider the credibility of witnesses or evaluate the weight of the evidence. A determination of the credibility of witnesses and the weight given to evidence must be made by the jury. Here, the court determined that the patient's two witnesses were not credible and gave no weight to their testimony in reviewing the motion for judgment as a matter of law. Thus, the court's ruling was improper. Answer choice A is incorrect. A motion for judgment as a matter of law may be made once a party has been fully heard on an issue at a jury trial. As in this case, the motion can be made at the close of the plaintiff's case. Answer choice C is incorrect. While the court must view the evidence in the light most favorable to the opposing party (the patient), the court's review of the evidence in this case was improper because it considered the credibility of witnesses. Answer choice D is incorrect because the jury, not the court, must make the determination as to credibility.

The driver of a pickup truck, through no fault of his own, struck a pedestrian who darted into the street in front of the driver's vehicle. The pedestrian was thrown up into the air and died instantly when her head struck the pavement of the street. The driver, panicked by the event, stepped on the accelerator pedal. The body of the pedestrian became lodged underneath the truck. The driver drove for a city block before abandoning the truck and fleeing on foot. Under state law, fleeing the scene of an accident is a felony. Can the driver be charged with involuntary manslaughter? A. No, because the driver acted in the heat of the moment without malice. B. No, because the pedestrian died when her head struck the pavement. C. Yes, because the driver was criminally negligent in driving for a city block after the striking the pedestrian. D. Yes, because the driver was guilty of a felony.

Answer Choice B. In order for a defendant to be guilty of involuntary manslaughter, there must be a causal connection between the unlawful act and the death. Here, although the pedestrian did die, and her death would not have occurred but for the pedestrian having been struck by the pickup truck, the facts provide that the driver was not legally responsible for that collision. The facts provide that he was not at fault for hitting the pedestrian, and it was the initial contact between the truck and pedestrian that caused her death. Answer choice A is incorrect. Although a defendant who is charged with murder can reduce the crime to voluntary manslaughter if the defendant committed the killing in the heat of passion, the crime of involuntary manslaughter is not similarly reduced to a lesser offense if the defendant acted in the heat of the moment. In addition, involuntary manslaughter is not avoided simply because the defendant does not act with malice. Answer choice C is incorrect. The driver's conduct in continuing to drive for a city block after striking the pedestrian certainly can be construed as criminal negligence. However, he acted in this manner after the pedestrian had already died through no fault of the driver's. Consequently, the driver's criminal negligence was not the cause of the pedestrian's death, and will not support a charge of involuntary manslaughter. Answer choice D is incorrect. Involuntary manslaughter can be based on the commission of a felony that is not a felony for purposes of the felony-murder rule, such as fleeing the scene of an accident. Here, however, the driver of the pickup truck was not in the process of committing or attempting to commit the felony at the time that the pedestrian was killed. He did not commit the felony of fleeing the scene of an accident until after the pedestrian was dead. Therefore, the felony will not be determinative of his liability for involuntary manslaughter.

Based on a recommendation by the local planning commission, the board of supervisors of a municipality enacted an ordinance that placed a maximum height restriction of two-stories on structures located on any oceanfront property. The ordinance, which was effective upon enactment, contained a provision that exempted existing residences from this new height limitation, but there were only a few existing three-story residences on oceanfront property in the municipality. An individual owned an oceanfront parcel. Prior to the adoption of the ordinance, he employed an architect to draft plans for a three-story residence on his property, the architect had completed the plans, and the homeowner had approved the plans and paid the architect. In addition, prior to the adoption of the zoning ordinance, the homeowner had applied for, but had not yet obtained, a building permit based on the plans. Has the homeowner acquired a vested right to have a three-story residence on his oceanfront property? A. No, because only a few existing residences qualified for the exemption. B. No, because the homeowner's request for a building permit had not been approved. C. Yes, because the homeowner had expended funds that directly related to building a three-story structure before the adoption of the change in the zoning ordinance. D. Yes, because the homeowner's demonstrable intention to build a three-story structure predated the adoption of the change in the zoning ordinance.

Answer Choice B. In order for a property owner who is in the process of building a structure when a zoning ordinance is adopted that prohibits such a structure to convince a court that his right to build the structure has vested, the property owner must have acquired a building permit prior to the date that the zoning ordinance took effect. Since this property owner was not awarded a building permit at the time that the zoning ordinance took effect, he does not have a vested right in a three-story residence on his oceanfront property. Answer choice A is incorrect because, while the number of existing structures that violate a zoning ordinance may be relevant in determining whether to issue a variance to a property owner, it is irrelevant to the determination of whether the property owner has acquired a vested right to build a structure that violates the zoning ordinance. Answer choice C is incorrect because a property owner's expenditure of funds that directly relates to the building of a structure is not in and of itself sufficient to create a vested right to do so in the face of a subsequent zoning ordinance that prohibits such a structure. Answer choice D is incorrect because a property owner's demonstrable intent to build a structure is not in and of itself sufficient to create a vested right to do so in the face of a subsequent zoning ordinance that prohibits such a structure.

A divorced man was considering entering his ex-wife's house and taking some rare books that the ex-wife had bought after the divorce. The divorced man thought that his portion of the couple's assets after the divorce had been inadequate, and that taking the books would make up the difference. Unsure about whether doing so was legal, he decided to consult an attorney. The attorney told the divorced man that, given the unfairness of the divorce settlement, it was fine to take the books. Late one night while she was out of town, the divorced man broke a window and entered his ex-wife's home. He took the rare books. A few days later, he was arrested for criminal trespassing. Can the divorced man raise his attorney's advice as a defense? A. No, because of the attorney-client privilege. B. No, because incorrect advice from an attorney is not itself a defense. C. Yes, because incorrect advice from an attorney is a defense. D. Yes, because his mistake of law was reasonable

Answer Choice B. Incorrect or bad legal advice from an attorney is not itself a valid mistake-of-law defense, but it may negate the required intent or mental state for a material element of the crime. Here, the divorced man consulted an attorney who told him that it was legal to take his ex-wife's rare books to make up for the unfairness of his divorce settlement. This is clearly incorrect legal advice. Thus, the divorced man cannot rely upon a mistake-of-law defense. For this reason, answer choice C is incorrect. Answer choice A is incorrect because the client, not the attorney, is the holder of the privilege. Thus, the attorney could not stop the divorced man from raising his advice as a defense. Answer choice D is incorrect. Even if the divorced man's mistake of law was reasonable, reliance on incorrect or bad legal advice from an attorney is not itself a valid mistake-of-law defense.

The owner of an older home sold the property to a buyer. The owner and the buyer conducted a reasonable inspection of the home prior to the sale and did not identify any dangerous conditions. A few days after the sale, the kitchen caught on fire while the buyer made dinner. The buyer was able to escape before the entire house burned down, but she suffered some burns and damage to her lungs from smoke inhalation. Afterwards, it was determined that an electrical fire had been caused by an unreasonably dangerous amount of faulty wiring behind the kitchen walls. The owner apologized to the buyer upon finding out about the fire, and truthfully told her that because the home had been unoccupied for many years, he had been unaware of the faulty wiring. Despite his apology, the buyer has sued him for negligence. Is the buyer likely to prevail in her action against the owner? A. No, because the owner conducted an inspection of the home with the buyer. B. No, because the owner was not aware of the faulty wiring. C. Yes, because the buyer was unlikely to have discovered the faulty wiring upon reasonable inspection. D. Yes, because the faulty wiring was an unreasonably dangerous condition.

Answer Choice B. Sellers of real property owe a duty to disclose to buyers those concealed and unreasonably dangerous conditions known to the seller. These are conditions that the buyer is unlikely to discover upon reasonable inspection. Here, the buyer was unlikely to discover the unreasonably dangerous condition of faulty wiring that was behind the kitchen walls during an inspection. However, because the owner was only obligated to disclose known conditions and he did not know about the wiring, the owner is not liable. Answer choice A is incorrect. Although the owner conducted an inspection of the home with the buyer, he would still be liable for any concealed and unreasonably dangerous conditions if they were known to him. An inspection alone would not release him of liability. Answer choice C is incorrect because the unlikelihood of discovery does not alter the owner's duty to disclose only known dangerous conditions. Answer choice D is incorrect because the fact that the faulty wiring was an unreasonably dangerous condition, while supporting the buyer's claim, is insufficient. The dangerous condition also needed to be known to the seller for the seller to be liable for a failure to disclose the condition.

At a jury trial for attempted murder, the prosecutor sought to introduce the testimony of the detective who had interrogated the defendant following his arrest. After many long and grueling hours of questioning, the detective obtained a confession from the defendant. The defendant contended that his confession was involuntary. Accordingly, the defendant sought a hearing outside the presence of the jury to determine the admissibility of the confession prior to the detective's testimony. Must the court grant the defendant's request? A. Yes, because the request involves a preliminary question of the competency of evidence. B. Yes, because the request involves the admissibility of a confession. C. No, because the decision to grant a hearing is within the court's discretion. D. No, because the admissibility of confessions is determined by the jury.

Answer Choice B. The trial judge generally decides preliminary questions regarding the competency of evidence, including the admissibility of evidence, whether a privilege exists, and whether a person is qualified to be a witness. Hearings on preliminary matters must be conducted outside the presence of the jury when the hearing involves the admissibility of confessions, when a defendant in a criminal case is a witness and so requests, or when justice requires it. In this case, the defendant sought a hearing outside of the presence of the jury regarding the admissibility of his confession. Accordingly, the court must grant the defendant's request and the judge will decide whether the confession is admissible. Answer choice A is incorrect. Hearings on preliminary matters must be conducted outside the presence of the jury only when the hearing involves the admissibility of confessions, when a defendant in a criminal case is a witness and so requests, or when justice requires it. Answer choice C is incorrect because the court must grant a defendant's request for a hearing outside the presence of the jury when it involves the admissibility of a confession. Answer choice D is incorrect because the trial judge generally decides the admissibility of evidence, such as a confession.

A bride brought an action for breach of contract against a radio DJ. The bride alleged that she entered into a signed written contract with the radio DJ to provide his services for her wedding reception, but he failed to provide entertainment at her wedding. The DJ has denied that he ever signed a contract with the bride. At the jury trial, the bride offered into evidence the purported written contract bearing the alleged signature of the radio DJ. The bride called the DJ's former business partner to testify that he became familiar with the DJ's signature while they worked together, and that the signature on the contract offered into evidence by the bride is the radio DJ's signature. The radio DJ objected to the business partner's testimony. How should the court rule? A. Overrule the objection, because a signature may only be authenticated by a person with personal knowledge of the signature. B. Overrule the objection, because the former business partner was familiar with the radio DJ's signature from working together. C. Sustain the objection, because the former business partner is not a qualified handwriting expert. D. Sustain the objection, because the authenticity of the signature on the contract must be determined by the jury.

Answer Choice B. The two methods by which handwriting verification may be used to authenticate a writing are: (i) comparison by an expert witness or the trier of fact, and (ii) the opinion of a lay witness familiar with the handwriting of the alleged author. For authentication by a lay witness, the lay witness must not have become familiar with the handwriting for the purposes of the current litigation. In this case, the business partner is a lay witness who had knowledge of the DJ's handwriting prior to the beginning of the litigation based on working with the radio DJ. Therefore, he can properly testify as to the radio DJ's signature. Answer choice A is incorrect. Although a signature may be authenticated by the testimony of a lay witness familiar with the handwriting of the alleged author, it may also be authenticated by an expert witness or the trier of fact. Answer choice C is incorrect because it is not necessary that the business partner be an expert in handwriting. He need only be familiar with the DJ's handwriting. Answer choice D is incorrect. Although the jury may compare the signature at issue with a genuine sample of the person's signature to authenticate a signature, this is not the only way that a signature may be authenticated. A signature may also be authenticated through the opinion of a lay witness familiar with the author's signature, as in this case.

A lumberjack decided to sell an old chainsaw he found in his shed. He could not remember when he had bought or last used it. Without testing the chainsaw, he cleaned it up so that it looked relatively new and posted a "For Sale" advertisement in the local newspaper. A woman who knew very little about chainsaws saw the lumberjack's advertisement, immediately contacted him, and went to see the chainsaw. When they met, the lumberjack told her that the chainsaw was in working order and was, in his opinion, one of the safest ones on the market. He also told her it was probably worth $1,000, but because she seemed like a nice lady, he was willing to give it to her for $200. Based on the lumberjack's statements, the woman purchased the chainsaw for $200. Later, when she tried to use the chainsaw, it would not turn on. The woman took the chainsaw to a repair shop. The shop told her the internal components of the chainsaw were rusted throughout, making it practically worthless. The shop charged her a discounted price of $800 to repair it to working order. The woman then sued the lumberjack for the tort of intentional misrepresentation. If successful, which of the following is the highest measure of damages the woman can recover? A. $800, because she can recover her out-of-pocket expenses to return the chainsaw to working order. B. $1,000, because her measure of recovery will be the "benefit of the bargain." C. Nothing, because the woman was under a duty to investigate the truth of the lumberjack's statements about the chainsaw. D. Nothing, because the woman's reliance on the lumberjack's statements about the chainsaw was not justifiable.

Answer Choice B. To recover for intentional misrepresentation, the plaintiff must establish a misrepresentation by the defendant, scienter, intent to induce the plaintiff's reliance, causation, justifiable reliance by the plaintiff, and damages. Here, the lumberjack made a material misrepresentation when he claimed that the chainsaw was in working condition without having tested it. Although he did not know that the chainsaw was actually broken, he acted with a reckless disregard as to the truthfulness of his representations to the woman by not checking first to see that it worked, especially because he was a lumberjack and thus had extensive experience with chainsaws. The lumberjack's misrepresentations induced the woman to buy the chainsaw, and her reliance on those misrepresentations was justifiable. In a majority of jurisdictions, the measure of recovery in misrepresentation cases is the "benefit of the bargain" rule, or the difference between the actual value received in the transaction ($0) and the value that would have been received if the misrepresentation were true ($1,000). Therefore, the woman will recover $1,000 as the benefit of the bargain. Answer choice A is incorrect because it does not apply the majority rule for calculating damages for the tort of intentional misrepresentation. Answer choice C is incorrect because a plaintiff bringing a tort for intentional misrepresentation is not required to investigate the truth of a defendant's statements. Answer choice D is incorrect because the woman was justified in relying upon the lumberjack's statements, as he was experienced in chainsaws, and she was not.

Deciding to expand her business, a high-end purse designer entered into a three-year lease with a factory owner to rent the factory for $200,000 per year. The written agreement included an option for the designer to purchase the factory from the owner for $1,000,000 before the end of the lease. The designer paid rent and made purses for three years. Prior to the expiration of the lease, the designer decided to purchase the factory pursuant to the option included in the lease agreement. For this reason, the designer purchased and installed new equipment so that she could expand her business to include making men's wallets. When the designer tried to exercise her option to purchase the factory at the end of the lease term, the owner refused to sell it to her, and forced the designer to leave the factory premises. The designer subsequently sued the owner to recover the reasonable value of the benefit conferred on the owner as a result of the installation of the new equipment. Which of the following is the strongest theory to support the designer's claim? A. Breach of an implied-in-fact promise B. Quasi-contract C. Conversion D. Breach of a resulting trust

Answer Choice B. When a plaintiff confers a benefit on a defendant and the plaintiff has a reasonable expectation of compensation, allowing the defendant to retain the benefit without compensating the plaintiff would be unjust. In this case, the court can permit the plaintiff to recover the value of the benefit to prevent the unjust enrichment. Thus, quasi-contract is the best theory for recovering the value of the benefit conferred on the owner as a result of the equipment installation, because the owner would be unjustly enriched. Answer choice A is incorrect. There is no implied-in-fact promise to compensate the designer for the equipment because retaking possession of the property does not imply a promise to compensate for any improvements to the property. Answer choice C is incorrect. The owner is permitted to take possession of the equipment, because the improvements are part of the property which is still owned by the owner. Answer choice D is incorrect. Based upon the fact pattern, there was no breach of a resulting trust because there was no creation of a trust or quasi-trust based upon the improvements.

The plaintiff, a State D citizen, is a shareholder of a corporation incorporated and having its principal place of business in State D. She brought an action in the U.S. District Court in State D requesting an injunction against the corporation, which allegedly was violating a State D law that required corporations to invest only in lawful commercial paper and that gave shareholders a right to sue over alleged violations of this law. The plaintiff claims that the defendant corporation had been purchasing commercial paper that had been issued by the federal government in violation of the U.S. Constitution. The corporation denies this claim and moves to dismiss on the ground that the U.S. District Court lacks jurisdiction. What would the court be most likely to do? A. Exercise general federal-question jurisdiction because federal law created the plaintiff's cause of action. B. Exercise general federal-question jurisdiction because the plaintiff has a real and substantial issue of federal law and her right to relief depends upon the resolution of this issue. C. Dismiss for lack of jurisdiction because the plaintiff's claim arose under state law because state law created her cause of action, and there is no diversity of citizenship. D. Dismiss for lack of jurisdiction because shareholders cannot sue their corporations in federal court, except for violations of federal securities law.

Answer Choice B. When state law creates a cause of action, a federal court can nonetheless exercise general federal-question jurisdiction if the complaint raises a real and substantial issue of federal law, and the outcome necessarily depends on resolving this federal issue. Here the plaintiff's complaint raises a genuine and substantial federal law issue (the constitutionality of the federal commercial paper), which must be resolved to decide her request for an injunction. Answer A is incorrect because the plaintiff's cause of action arises under state law, which requires corporations to invest only in lawful commercial paper. Answer C is incorrect because, even though state law created the plaintiff's cause of action and diversity jurisdiction does not exist, federal-question jurisdiction may be invoked. Answer D is incorrect because there is no rule prohibiting a shareholder from suing her corporation in federal court.

The owner of an office building leased office space to a realtor for a term of five years. The lease contained a provision that prohibited the assignment of the lease without the owner's permission. After both parties complied with the terms of the lease for 18 months, the realtor leased the premises to an accountant for a one-year period. The realtor did not seek the permission of the owner. The owner accepted payment of the monthly rent from the accountant without objection during the one-year period. Without seeking the owner's permission, the realtor then entered into a written agreement to lease the premises to a lawyer for the remaining 30 months of the five-year period. If the owner sues the realtor for breach of their lease agreement, is he likely to prevail? A. No, because the realtor is no longer in privity of estate with the owner. B. No, because the owner waived the provision in the lease prohibiting assignment by accepting rental payments from the accountant. C. Yes, because the realtor failed to seek the owner's permission to lease the office space to the lawyer. D. Yes, because the realtor was prohibited from leasing the office space to the lawyer.

Answer Choice C. An assignment is a complete transfer of the tenant's remaining lease term. In this case, the lease between the owner and realtor contained a provision that prohibited the assignment of the lease without the owner's permission. The realtor breached this provision by entering into a lease with the lawyer for the remaining 30-month term of the owner-realtor lease, creating an assignment. Consequently, the owner can bring suit against the realtor for breach of their lease agreement. Answer choice A is incorrect. The realtor is no longer in privity of estate with the owner because the realtor has entered into an assignment of the realtor's interest in the premises. However, the realtor remains in privity of contract with the owner and is subject to the lease agreement that he entered into with the owner. Answer choice B is incorrect. Although the owner did accept rental payments from the accountant, the accountant was not an assignee, but only a sublessee because the accountant rented the office space from the realtor for only a one-year period rather than for the full remaining term of the lease between the realtor and the owner. Consequently, the owner's action in accepting rental payments from the accountant did not constitute a waiver of the assignment-restriction provision. Answer choice D is incorrect. Although the lease provision prohibited the realtor from leasing the office space to the lawyer without the owner's permission, this provision does not bar the realtor from doing so, but instead makes the realtor's action a breach of the owner-realtor lease.

An individual who was a U.S. citizen but had permanently moved to a foreign country returned to the United States for an extended visit with relatives who lived in State A. While in State A, the individual suffered serious injuries when an amusement park ride malfunctioned. The individual filed a negligence action in federal district court seeking $95,000 against the corporation that owned the amusement park in which the ride was located. The corporation is incorporated in State B and has its principal headquarters in State A. The corporation has filed a motion to dismiss for lack of subject matter jurisdiction. Should the court grant the motion to dismiss? A. Yes, because the corporation is a citizen of the same state where the cause of action arose. B. Yes, because the individual's domicile is in a foreign country. C. No, because both the individual and the corporation are U.S. citizens who are not citizens of the same state. D. No, because the corporation is only a citizen of the state in which is incorporated.

Answer Choice B. When the cause of action is based on state common law rather than a federal question as is the case here, subject matter jurisdiction exists on the basis of diversity of citizenship when the plaintiff and the defendant are U.S. citizens as well as citizens of different states. Here, although the individual and the corporation are both U.S. citizens, and the corporation is a citizen of both State A where its principal headquarters is located and State B where it is incorporated, the individual is not a citizen of any U.S. state, but instead, having moved permanently to a foreign country, is domiciled in the foreign country. Therefore, there is no diversity of citizenship because the action is not between citizens of different States. Moreover, alienage jurisdiction is lacking because the individual is not an alien: the facts indicate that the individual has retained his U.S. citizenship. Consequently, subject matter jurisdiction does not exist. Answer choice A is incorrect because, unlike venue, subject matter jurisdiction does not turn on where the cause of action arose. Answer choice C is incorrect because, even though the individual and the corporation are not citizens of the same state, diversity of citizenship does not exist when, even though both the plaintiff and the defendant are U.S. citizens, one party is not a citizen of any U.S. state. Answer choice D is incorrect because a corporation is a citizen where its principal headquarters is located as well as a citizen of the state of its incorporation. In addition, the court should grant the motion to dismiss because, since the individual is not a citizen of any U.S. state, diversity jurisdiction does not exist.

The defendant and his friend rented a house together. Although both signed the lease and each paid half of the monthly rent, each had exclusive control over his own bedroom. Two police officers with probable cause that the house contained stolen large-screen televisions sought permission from the friend to search for them. The friend, certain that there were no such televisions on the premises, voluntarily told the officers that they could search the entire house for the televisions. The officers did not warn the friend that he had the right to refuse to consent to the search. Reasonably relying on the friend's permission, the officers searched the house and found two stolen televisions in the defendant's bedroom under his bed. The defendant was charged with theft and possession of the stolen televisions. The defendant moved to exclude the televisions as evidence. Will the defendant's motion to suppress be granted? A. Yes, because the police failed to warn the friend of his right to refuse to consent to the search. B. AYes, because the friend had no authority to grant permission for the search of the defendant's bedroom. C. No, because the police reasonably believed that the roommate had the authority to grant permission for the search. D. No, because the police had probable cause to search the house.

Answer Choice C. A third party's consent to a search is not invalid because he lacks the authority to consent, so long as the police reasonably believe that he does have such authority. Answer choice A is incorrect because an otherwise voluntary consent is not void merely because the police do not warn the person that he has the right to refuse to consent. Answer choice B is incorrect because, even though the friend lacked actual authority to consent to the search of the defendant's bedroom, the police reasonably believed he had such authority. Answer choice D is incorrect because probable cause alone is not sufficient to support a warrantless search of a residence; one of the warrant exceptions, such as consent, must apply.

A bank teller is on trial for embezzling funds from a school parent-teacher association on which he serves. The teller seeks to call his former boss as a witness at the embezzlement trial, who will testify that the teller, on five separate occasions over a two-month period during the previous year, alerted bank customers to errors in their cash deposit slips that saved the customers money. The teller wants to offer this testimony to show his character for truthfulness and does not seek to testify or to offer any further evidence of his good character at trial. Is the former boss's testimony admissible? A. Yes, because a criminal defendant may submit evidence to show his good character so long as it is relevant to an element of the crime charged. B. Yes, because evidence of specific acts of a criminal defendant's good character is admissible so long as they do more than simply bolster the defendant's credibility. C. No, because evidence of specific acts of a defendant's good character is not admissible in criminal cases. D. No, because character evidence is not admissible to prove that a person acted in conformance with that character on a particular occasion.

Answer Choice C. A criminal defendant is permitted to introduce evidence of his good character as being inconsistent with the type of crime charged, but that evidence must be (i) pertinent to the crime charged and (ii) in the form of reputation or opinion testimony. Evidence of specific acts of the defendant's good character is not admissible. In this case, the former boss's testimony concerns specific acts of the defendant's good character. Therefore, the testimony is not admissible. Answer choice A is incorrect. A criminal defendant may introduce evidence of his good character if it is relevant to an element of the crime charged, but must do so with reputation or opinion testimony and not testimony regarding specific acts. Answer choice B is incorrect. The former boss's testimony is not an admissible form of character evidence because it concerns specific acts of the defendant, even if it doesn't simply bolster the defendant's credibility. Answer choice D is incorrect. While it is true that character evidence is generally not admissible to prove conformance with that character on a particular occasion, the defendant in this case is submitting the testimony of his former boss to prove his character for truthfulness. Criminal defendants may submit reputation or opinion testimony of their good character if it is relevant to an element of the crimes charged. Here, the teller's truthfulness is relevant to an element of embezzlement, which requires that the defendant act fraudulently.

The plaintiff, a resident of State A, was visiting the home of the defendant, who resides in State B. As the plaintiff was leaving the house, she fell through a rotted plank in the front steps and broke her ankle. The plaintiff underwent several surgeries and extensive rehabilitation. She brought suit in federal district court, alleging that the defendant negligently maintained her premises and asking for relief in the amount of $150,000. The defendant filed a counterclaim alleging that the plaintiff owes her $5,000 as payment for web design work that the defendant completed for the plaintiff. If the plaintiff filed a motion to dismiss the defendant's counterclaim for lack of jurisdiction, will the plaintiff succeed? A. No, because the parties maintain complete diversity. B. No, because the federal district court may exercise supplemental jurisdiction over counterclaims. C. Yes, because the defendant's counterclaim does not meet the amount-in-controversy requirement for diversity jurisdiction. D. Yes, because the federal district court may only exercise supplemental jurisdiction when the action includes compulsory counterclaims.

Answer Choice C. A federal court may assert supplemental jurisdiction over a counterclaim that does not satisfy the jurisdictional amount when the counterclaim is compulsory. However, a permissive counterclaim does not qualify for supplemental jurisdiction and therefore must satisfy the jurisdictional amount and the rule of complete diversity. Here, the defendant's claim that the plaintiff owes her money does not arise out of the same transaction and occurrence as the plaintiff's injuries. Therefore, the defendant's counterclaim is permissive and must meet the amount-in-controversy requirement for diversity jurisdiction. Answer choice A is incorrect. Although the parties must maintain complete diversity, a permissive counterclaim must also meet the amount-in-controversy requirement. Answer choice B is incorrect. Although the federal district court may exercise supplemental jurisdiction over counterclaims, a permissive counterclaim needs to meet the requirements for federal subject-matter jurisdiction (either diversity or federal question). Answer choice D is incorrect. While it is true that the defendant's counterclaim is not compulsory, it would be possible for the federal court to assert diversity jurisdiction had the defendant met the amount-in-controversy requirement. Therefore, D is not the best answer choice.

After the plaintiff properly filed a breach of contract action in federal district court, the defendant timely filed a demand for a jury trial. The defendant then timely filed a summary judgment motion. The plaintiff filed a response in opposition to the motion for summary judgment. In ruling on the defendant's motion for summary judgment, the court should apply which of the following standards? A. The court must grant the motion if the jury, based on a preponderance of the evidence, would find that the defendant is entitled to judgment. B. The court may grant the motion if the jury, based on a preponderance of the evidence, would find that the defendant is entitled to judgment. C. The court must grant the motion if there is no genuine dispute as to any material fact and the defendant is entitled to judgment as a matter of law. D. The court may grant the motion if there is no genuine dispute as to any material fact and the defendant is entitled to judgment as a matter of law.

Answer Choice C. A motion for summary judgment must be granted if there is no genuine dispute as to any material fact and the moving party is entitled to judgment as a matter of law. Answer choices A and B are incorrect because they do not state the correct standard. The standard for granting a summary judgment motion applies to all civil actions; it does not vary depending on whether a jury trial has been demanded by a party. Moreover, answer choice B is further incorrect because if the standard is met, the court must grant the motion. Answer choice D is incorrect because, while the standard for granting a summary judgment motion is that there is no genuine dispute as to any material fact and the moving party is entitled to judgment as a matter of law, the court must grant the motion if this standard is met.

A manufacturer of paper bags had a contract with a retail grocer to supply the grocer with paper bags on a quarterly basis. The grocer had always complied with the payment terms of the contract, which give the grocer a month after receipt of the paper bags to pay for them. The manufacturer heard a rumor that the grocer had consistently failed during the past several months to timely pay a frozen food company for merchandise delivered to the grocer. The rumor was false, but led the manufacturer, as well as other companies that supplied goods to the grocer, to reasonably question the retailer's ability to pay for their goods. Forty-five days before the next shipment of paper bags was due, the president of the manufacturer called the grocer and demanded that the grocer provide assurances that it could pay for the paper bags. The grocer did not give the manufacturer assurances before the next shipment of paper bags was due. If the manufacturer fails to make the next shipment, which of the following provides the grocer with the best argument that the manufacturer is in breach of the contract? A. The manufacturer cannot demand assurances because the grocer has not breached the contract. B. The manufacturer has not given the grocer sufficient time in which to provide assurances. C. The manufacturer's demand for assurances was not in writing. D. The rumor about the grocer's inability to pay for goods on time was false.

Answer Choice C. A party can demand assurances of performance if there are reasonable grounds for insecurity about the other party's ability or willingness to perform. Once such assurances are requested, performance may be suspended until they are provided. Failure to give adequate assurances within a reasonable time can be treated as a repudiation. Under the UCC, the demand for assurances must be made in writing. Here, the contract for paper bags is a contract for the sale of goods that is governed by the UCC. Accordingly, the manufacturer's failure to make its demand for assurances in writing could deny the manufacturer its right to suspend its performance based on that demand. Answer choice A is incorrect because the right to demand adequate assurances need not be based on a party's past performance of its obligations under the contract with the party demanding assurances. It can also arise from other circumstances, such as the party's failure to perform its obligations under a contract with a third party. Therefore, the grocer's prior performance of its obligations under its contract with the manufacturer is not certain to protect the grocer from a demand by the manufacturer for adequate assurances. Answer choice B is incorrect because, under the UCC, the outer limit for a reasonable time in which to give adequate assurances is 30 days. Here, the grocer did not respond to the manufacturer's demand for 45 days. Answer choice D is incorrect because, even though the rumor is false, the manufacturer's questioning of the grocer's ability to pay is reasonable. Moreover, other suppliers of goods to the grocer also questioned the grocer's ability to pay for the goods. Between merchants, the reasonableness of the grounds for insecurity is determined according to commercial standards. Thus, the manufacturer had the right to demand adequate assurances.

The owner of a building in a large city wanted to start a business of horse-drawn carriages to give tours. After determining that his plan was permitted by all applicable zoning codes, he converted the building into code-compliant horse stables. He bought a number of horse-drawn carriages that he kept in the building's garage, and obtained 30 horses from a horse rescue mission. Although the stable conditions were compliant with all relevant codes, the unavoidable smells of any horse stable began to permeate throughout the neighboring apartment buildings. After the owner operated his stables for a year without his neighbors voicing any complaints, a new tenant moved into a neighboring apartment building. Although the tenant knew about the stables before moving in, he brought a public nuisance action against the owner, citing the stench. Which of the following is the owner's strongest defense against the neighboring tenant's claim? A. No other neighbors have complained about the stables for a year. B. The converted stables comply with all local zoning and safety codes. C. The tenant has not suffered a harm different from that suffered by the other nearby residents. D. The tenant moved next door knowing about the owner's horses and stable operation.

Answer Choice C. A public nuisance is an unreasonable interference with a right common to the general public. However, a private citizen has a claim for public nuisance only if she suffers harm that is different in kind from that suffered by members of the general public. Because there is no evidence that the new tenant is suffering a different sort of injury than his neighbors, a public nuisance action is the improper claim to bring. Therefore, this is the owner's strongest defense. Answer choice A is incorrect. The fact that no one has complained yet does not conclusively establish that an interference is not a substantial and unreasonable interference. Therefore, although this is supported by the fact patterns, this is not an effective defense against the tenant's claim. Answer choice B is incorrect because the fact that a defendant complies with a statute, local ordinance, or administrative regulation is not a complete defense to a nuisance action. Answer choice D is incorrect because it is generally not a defense that the plaintiff "came to the nuisance" by purchasing property in the vicinity of the defendant's premises with knowledge of the nuisance operated by the defendant.

A state statute provides that only residents of an incorporated city may vote in elections for city officials. An owner of a small business located in an incorporated city cannot vote for city officials because the owner does not reside within the city limits. The owner asserts that he has a vested interest in the choice of city officials because he is subject to licensing fees required by city ordinances in order to do business in the city. The owner has brought an action challenging the constitutionality of the residency requirement. Is the residency requirement constitutional? A. No, because it discriminates against a person who is directly affected by decisions rendered by city officials. B. No, because this restriction is not the least restrictive means to achieve a compelling interest. C. Yes, because it is rationally related to a legitimate state purpose. D. Yes, because voting in a local election is not a fundamental right.

Answer Choice C. A restriction on the right to participate in the political process of a governmental unit to those who reside within its borders is typically upheld as justified on a rational basis. Nonresidents generally may be prohibited from voting. Answer choice A is incorrect because the restriction on the right to participate in the political process of a governmental unit, including a city, may preclude those who do not reside within its borders, even though they may be directly affected by decisions made by city officials, such as a licensing fees imposed on those who operate a business within the city. Answer choice B is incorrect because, although voting is a fundamental right, not all restrictions on the right to vote are subject to strict scrutiny. This restriction based on residency within the governmental unit is only subject to rational basis review. Answer choice D is incorrect because voting is a fundamental right, even in a local election.

In a retailer's strict products liability action against a manufacturer, the retailer sent the manufacturer a properly served interrogatory asking for information about the manufacturing process it used in making its widgets. The manufacturer replied by identifying the process patent that detailed its manufacturing process, claiming that it followed this process to manufacture its widgets. The retailer wants to admit a properly authenticated copy of the process patent into evidence to establish a possible design defect. Is the copy of the patent admissible? A. No, because it is hearsay not within any exception. B. No, because the copy of the patent violates the best evidence rule. C. Yes, as an adoptive admission by the manufacturer. D. Yes, as a public record.

Answer Choice C. Answers to interrogatories, which are signed and made under oath by a party to the litigation, are nonhearsay admissions. Because an interrogatory by the manufacturer included by reference the content of the identified patent, the patent will also be admissible as an adoptive admission by the manufacturer. Answer choice A is incorrect because, as an adoptive admission, the patent is nonhearsay. Answer choice B is incorrect. Even if the contents of the patent are considered to be at issue in this case, a duplicate is admissible to the same extent as an original unless there is a genuine question as to the authenticity of the original, or the circumstances make it unfair to admit the duplicate. Therefore, there is no reason on these facts that the admission of the copy of the patent would violate the best evidence rule. Answer choice D is incorrect. Although a patent could arguably fall into the public record hearsay exception, because the patent here is an adoptive admission by the manufacturer, it is non-hearsay. Therefore, no hearsay exception is necessary, and the public record hearsay exception is irrelevant.

The defendant was tried for armed robbery. The state introduced evidence that a man, identified by witnesses as the defendant, entered a convenience store at 11 pm on March 5, threatened the clerk with a gun, and took $75 from the cash register. The defendant did not testify, but his sister did. She testified that on march 5, at the time of the robbery, the defendant was with her in a city 300 miles away. On cross-examination, the sister admitted having given a statement to the police in which she had said that the defendant was not with her on March 5, but she claimed that the earlier statement was mistaken. The court instructed the jury that in order to convict the defendant, they had to find all of the elements of the crime beyond a reasonable doubt. As to the defendant's claim of alibi, which of the following additional instructions would be proper? (A) Alibi is a matter of defense and so must be established by the defendant; however, the burden of persuasion is by the preponderance of the evidence, not beyond a reasonable doubt. (B) Before you may consider the defendant's claim of alibi, you must decide whether he has produced sufficient evidence to raise the issue. (C) If you have a reasonable doubt as to whether the defendant was present at the convenience store at about 11 pm on March 5, you must find him not guilty. (D) If the defendant's evidence has caused you to have a reasonable doubt as to whether he was the robber, you must find him not guilty.

Answer Choice C. Because the defendant's present at that time and place was an element of the crime charged, as to which the prosecution bore the burden of proof beyond a reasonable doubt. The Supreme Court has long held that the Due Process Clause protects the accused against conviction except upon "proof beyond a reasonable doubt of every fact necessary to constitute the crime with which he is charged." Given the prosecution's theory of the case here, the defendant's presence at the convenience store at 11 PM on March 5 was a "fact necessary to constitute the crime charged." Therefore, the prosecution was required to prove that fact beyond a reasonable doubt. Since this instruction carefully declines to place any burden at all on the defendant with regard to the alibi defense, and all of the other instruction choices impose some burden of proof on the defendant regarding that defense, this choice is the only correct one.

In response to a request by the president of a local sorority, the owner of a liquor store agreed to supply beer to the sorority for a party at a 10 percent discount off the regular price. The owner was aware that a number of sorority members were under the age of 21, but he did not intend to supply alcoholic beverages to individuals under the age of 21. The owner delivered the beer to the sorority and the president of the sorority, who was 21 years old, paid for the beer. The sorority permitted anyone who attended the party, including those under 21 years of age, to drink the beer supplied by the owner. By statute, it is a criminal offense if a person supplies an alcoholic beverage to an individual under 21 years of age. This statute has been interpreted as a strict-liability offense. Can the owner properly be charged with conspiracy to supply alcoholic beverages to individuals under the age of 21? A. Yes, because the owner should have been aware that there was a substantial and unjustifiable risk that the beer would end up in the hands of individuals who were under the age of 21. B. Yes, because the offense of supplying beer to individuals under age 21 has been interpreted as a strict-liability offense. C. No, because the owner did not have the specific intent to supply alcoholic beverages to individuals under the age of 21. D. No, because a merchant of goods cannot be criminally liable for legally selling such goods.

Answer Choice C. Conspiracy is a specific-intent crime. Consequently, even though the offense of supplying alcoholic beverages to persons under the age of 21 is a strict-liability offense, conspiracy to commit this offense is a specific-intent crime. Because the owner did not have the specific intent to supply alcoholic beverages to individuals under the age of 21, the owner cannot properly be charged with conspiracy. Answer choice A is incorrect. Although the owner arguably should have been aware that there was a substantial and unjustifiable risk that the beer would end up in the hands of individuals who were under the age of 21, the owner's negligence, which would be more than sufficient to convict the owner of the offense of supplying beer to individuals under the age of 21, is not sufficient to charge the owner with conspiracy to commit this offense. Answer choice B is incorrect because, even though the offense of supplying alcoholic beverages to persons under the age of 21 is a strict-liability offense, conspiracy to commit this offense is a specific-intent crime. Answer choice D is incorrect because it misstates the law. Because conspiracy requires not only an intent to agree but also an intent to commit the crime, criminal liability for conspiracy cannot be based solely on knowledge of the existence of the conspiracy. Here, the owner who supplied beer to the sorority arguably knowing that the sorority likely intended to supply the beer to underage individuals is not a member of the conspiracy simply because the owner may have possessed such knowledge. Because the owner did not take an additional step to show such intent, such as selling the beer at an exorbitant price (here, the owner sold the beer at a 10 percent discount to the sorority), the owner cannot properly be charged with conspiracy.

A husband and his business partner owned a large technology company together. After a personally tumultuous but professionally successful decade working together, the husband discovered that the business partner had been fraudulently transferring company funds to the business partner's personal account for years. Before he confronted his business partner about this, he called his own wife to tell her what he had learned. His wife reminded him that the company had a $2 million life insurance policy on the business partner. The couple formed a plan to murder the business partner for the insurance proceeds when he was alone in the office building. On the day that they planned to carry out the murder, the husband told the business partner that he had to leave early and asked the business partner to stay late to finish up a presentation. He knew that by doing so, the business partner would be alone at the office. Later that night, the wife went in and shot the business partner. She then panicked and fled the country. The husband was later charged with the murder and conspiracy to commit the murder, but the wife was never apprehended. The jurisdiction recognizes the majority rule regarding conspirator liability. Is the husband likely to be found guilty of conspiracy and murder? A. No, because the husband and wife were married. B. No, because the wife was never convicted of the murder. C. Yes, because the husband persuaded the business partner to stay late at the office. D. Yes, because no overt act was required in furtherance of the crime.

Answer Choice C. Conspiracy is an agreement between two or more persons to accomplish an unlawful purpose with the intent to accomplish that purpose. The majority rule requires an overt act in furtherance of the conspiracy, although the common law does not. A conspirator can be convicted of both the offense of conspiracy and all substantive crimes committed by any other co-conspirators acting in furtherance of the conspiracy. In this case, the husband and wife formed a plan to murder the business partner, and the husband committed an overt act when he asked the business partner to stay late one night to carry out the murder. Answer choice A is incorrect because, although the common law did not consider husband and wife as co-conspirators, nearly every jurisdiction has abolished this common-law concept. Answer choice B is incorrect because, although a conspirator cannot be convicted of conspiracy if all other conspirators are acquitted at the same trial, a co-conspirator can be convicted if other co-conspirators are never tried or apprehended. Answer choice D is incorrect because the husband did commit an overt act when he persuaded the business partner to stay late at the office.

A plaintiff filed a complaint alleging malpractice in federal court. The plaintiff properly served the defendant with the complaint and summons. The defendant timely served its answer together with a set of 24 interrogatories and a request for a Rule 26(f) conference on the plaintiff. The interrogatories provided that the plaintiff must respond to the interrogatories within 30 days of service. The interrogatories sought disclosure of facts relating to the plaintiff's claim and in addition, asked the plaintiff to state the facts upon which he intended to rely at trial to support his allegation that the defendant breached the applicable standard of care. The plaintiff has moved for a protective order from the court regarding the interrogatories. Which of the following provides the strongest support for the plaintiff's motion? A. The interrogatories asked about the plaintiff's factual and legal contentions. B. The number of interrogatories exceeds the number permissible without court permission or the plaintiff's consent. C. Service of the interrogatories is premature. D. The rules permit the plaintiff to take longer than 30 days respond to the interrogatories.

Answer Choice C. In general, a party may not serve interrogatories before the parties have held a Rule 26(f) discovery conference. Since this conference has not been held, the defendant's interrogatories are premature. Answer choice A is incorrect because an interrogatory is not objectionable merely because it asks for an opinion or contention that relates to fact or the application of law to fact. Answer choice B is incorrect because, since any party may serve no more than 25 written interrogatories on any other party, the number of interrogatories served by the defendant is not improper. Answer choice D is incorrect because, in the absence of a court order or an agreement by the parties, the responding party must serve its answers and any objections within 30 days after being served with the interrogatories. Consequently, the 30-day response period is correct.

A homeowner in a common-interest community planned to build a deck on the back of his house that overlooked the community lake. As required by the homeowner's deed, as well as the deeds of the other community members, the homeowner submitted his plans to the community association's architectural review board prior to construction for the board's approval. The board, with express power in all of the property deeds of the community to reject a plan "on any ground, including aesthetics, in its sole and uncontrolled discretion," rejected the plans for the deck, finding that the deck was substantially larger than other structures approved by the board and built by other community members. The board subsequently asked the homeowner to resubmit plans for a smaller deck. The homeowner refused to do so and began construction on the rejected deck. The association filed a lawsuit seeking an injunction to prevent the homeowner from building the planned deck. Is the court likely to grant the association's requested injunction? A. No, because a common-interest community association cannot impose aesthetic controls on a homeowner's use of his property. B. No, because there were no standards imposed on the architectural review board's exercise of its discretion. C. Yes, because the architectural review board acted reasonably in exercising its review power. D. Yes, because the architectural review board has unlimited discretion to reject the plans for any structural improvement.

Answer Choice C. In the exercise of its discretionary powers, a common-interest community association or one of its bodies, such as an architectural review board, must act reasonably. Here, the review board made its decision to reject the homeowner's deck on the basis of its size in comparison to other structures built by other community members. Consequently, the court is likely to find that the board's rejection of the homeowner's plans was reasonable, and will most likely grant the association's request for an injunction. Answer choice A is incorrect because, while a common-interest community association, or one of its bodies, does not automatically possess the right to impose design restrictions on a member's use of her property, the deed gave the architectural review board such power. Answer choice B is incorrect because, while the grant of review authority to the architectural review board did not impose specific standards on the board in the exercise of its authority, the board in this instance did exercise such review in a reasonable manner as required by law. Answer choice D is incorrect because, despite the wording of the board's discretionary power of review, the board must act reasonably in exercising its power of review.

One afternoon, a woman was walking down the street when a man walked up behind her, grabbed her purse, and ran away. That evening, as the woman and her husband dined at a restaurant, she recognized an identical purse hanging from the back of a chair upon which another woman was sitting at a table a few feet away. She pointed the purse out to her husband before going up to the table to confront the couple sitting there. The couple claimed that the purse was certainly not stolen and refused to give it to her, so the woman violently yanked it from the chair, knocking it and the woman sitting on it over, and returned to the table with her husband. Robbery charges were subsequently brought against her, and during trial, evidence indicated the purse did actually belong to her. Is the woman likely to be successfully convicted of robbery? A. Yes, because she stole the property at night. B. Yes, because she took the purse by force. C. No, because the purse had been stolen from her earlier in the day. D. No, because she took the purse from the chair, not the other woman.

Answer Choice C. Robbery is larceny from the person or presence of the victim by force or intimidation. The force used by the defendant must be more than the amount necessary to effectuate taking and carrying away the property. Here, the woman did use force to take the property from another, with the intent to permanently deprive her of that property, but because the property was hers to begin with, she did not take property that belonged to another. Answer choice A is incorrect because robbery need not occur at night. The answer alludes to the requirements for common law burglary. Answer choice B is incorrect because while the woman did use force to obtain the purse, that is only one element of robbery, not all of which are met here. Answer choice D is incorrect because, while the force need not be applied to the actual person if the other elements of robbery were met, those requirements were not met here.

Due to the scarcity of post offices in low-income inner-city communities, the few post offices in the communities suffered severe overcrowding, slowing postal service to and from these communities significantly. To address the problem, Congress enacted a statute authorizing the conversion of old, uninhabited buildings in these communities that were owned by the government into post offices. A historical society in one of the affected communities wants to stop Congress from converting the buildings into post offices because they were categorized as historical buildings of cultural value to the community. Is the statute constitutional? A. No, because establishing postal services is a power reserved to the states. B. No, because the statute violates the Takings Clause. C. Yes, under Congress's property power. D. Yes, because the properties will be converted for a public use.

Answer Choice C. The Federal Property Clause of Article IV, Section 3 gives Congress the "power to dispose of and make all needful rules and regulations respecting the territory or other property belonging to the United States." In addition, there is no express limit on Congress's power to dispose of property owned by the United States. Here, Congress has the power to make all needful rules respecting the old buildings owned by the government, including converting them into post offices. Answer choice A is incorrect because it misstates the law. Congress, not the states, has the exclusive power "to establish post offices and post roads" under Article I, Section 8, Clause 7. Answer choice B is incorrect because the Takings Clause does not apply when the government is putting property owned by the government to a use that does not rise to the level of a regulatory taking of surrounding property. Answer choice D is incorrect because there is no limitation under Article IV, Section 3, that Congress can only use properties owned by the government for a public use.

A prisoner at a federal prison contracted a staph infection that was resistant to the antibiotics used to treat ordinary staph infections. The warden refused to provide the prison doctor with the funds necessary to purchase the appropriate antibiotics for the prisoner's infection. As a result, the prisoner died. The prisoner's wife filed a wrongful death suit against the warden. At trial, the prison doctor was called to testify that the prisoner was not given the appropriate antibiotics. During her testimony, the prison doctor made general statements about the lack of appropriate medical care in prisons, and the need for improvement for the health and safety of the prisoners. After the trial, the prison doctor was fired by the warden for the statements she made during the trial about the lack of appropriate medical care in prisons. The prison doctor subsequently sued the warden in his official capacity, alleging that the warden violated her First Amendment rights by firing her in retaliation for her testimony. Did the warden violate the prison doctor's First Amendment rights? A. No, because the prison doctor, as a government employee, is prohibited from publicly commenting on government policies. B. No, because the prison doctor's statements were made within the scope of her official duties. C. Yes, because government employees are entitled to the same free speech protection as other citizens. D. Yes, because the prison doctor was speaking as a citizen on a matter of public concern.

Answer Choice D. When a government employee contends that her rights under the Free Speech Clause of the First Amendment have been violated by her employer, the employee must show that she was speaking as a citizen on a matter of public concern. When a government employee is speaking pursuant to her official duties, the employee is generally not speaking as a citizen and the Free Speech Clause does not protect the employee from employer discipline. In determining whether a government employee is speaking pursuant to her official duties, the critical question is whether the speech at issue is itself ordinarily within the scope of an employee's duties, not whether it merely concerns those duties. Here, the prison doctor is a government employee. Although she was testifying regarding the fact that the prisoner did not receive the proper medication for his infection, she also made general statements about the healthcare system in prisons. In making these general statements, she was speaking as a citizen on a matter of public concern, even though the statements concerned information about prison conditions that was learned through her employment. Therefore, the statements for which the prison doctor was fired were not made within the scope of her official duties. Additionally, the First Amendment interest of the employee must be balanced against the interest of the state, as an employer, in effective and efficient management of its internal affairs. Here, the prison doctor's interest in speaking freely about the overall problems with the prison healthcare system likely outweighs the interest of the state as an employer, as the doctor's general statements are not likely to disrupt the state's management of its affairs. Instead, the statements merely shed light on a relevant issue. Answer choice A is incorrect. Government employees do not give up their right to free speech solely by virtue of being a government employee. Government employees are permitted to publicly comment on government policies and such comments will be protected if the employee was speaking as a citizen on a matter of public concern, as in this case. Answer choice B is incorrect because, as discussed above, the prison doctor was not speaking pursuant to her official duties when she made the general statements about the lack of appropriate medical care in prisons. Answer choice C is incorrect. Although a government employee is entitled to the same free speech protection as other citizens when speaking as a citizen on a matter of public concern, a government employee is not entitled to such protections if the employee is speaking pursuant to her official duties.

On the basis of an eyewitness description, a student was arrested in connection with the murder of a security guard who was killed during a protest at a college. The police brought the student to the police station and gave him his Miranda warnings. The student then asked to speak with his attorney. Before the attorney could arrive, the eyewitness walked into the station, saw the student, and told the police that this was the person she had seen kill the security guard. The prosecution sought to introduce at trial evidence of the eyewitness identification. The defense moved to have evidence of the identification suppressed. The court should: A. Grant the motion because the student's Sixth Amendment rights were violated. B. Grant the motion because this was a corporeal identification procedure. C. Deny the motion because there was no Sixth Amendment violation. D. Deny the motion because this was a non-corporeal identification procedure.

Answer Choice C. The Sixth Amendment right to counsel does not apply to any pre-indictment eyewitness identification. This identification was pre-indictment and therefore the motion to suppress should be denied. Answer choice A is incorrect for the reason just stated, that this was a pre-indictment identification to which the Sixth Amendment does not apply. Answer choice B is incorrect because even though this was a corporeal identification procedure in that it was an in-person identification, it occurred pre-indictment. Answer choice D is incorrect factually, as this was in fact a corporeal identification.

A father who owned land in fee simple absolute devised the land by will as follows: "To my daughter, her heirs and assigns; but if my son is alive twenty-five years from the date of my death, then to my son, his heirs and assigns." Shortly after the father's death, the son learned that he has a terminal illness. Currently, the son is alive and the daughter has given birth to a child. The jurisdiction continues to adhere to the common law Rule Against Perpetuities. Which of the following is the best argument that the daughter currently owns the land in fee simple absolute? A. The daughter has an heir. B. The son will die before his interest becomes possessory. C. There can be no future interest after the grant of a fee simple absolute. D. The son's interest could not have become possessory within twenty-one years after his father's death.

Answer Choice C. The daughter's best argument is that, reading her father's devise from left to right ("to my daughter, her heirs and assigns"), he granted her the land outright without any conditions. Having done so, he could not then place conditions on that grant. Answer choice A is incorrect because use of the phrase "her heirs and assigns" is an accepted, but no longer required way in which the grantor can indicate his intent to convey a fee simple absolute interest in the property. The grantee is not required to have heirs (or assigns) in order to take the property in fee simple absolute. Answer choice B is incorrect because, if the daughter's argument is rejected, then the son has an executory interest in the land. Although upon the son's death his executory interest will cease to exist, currently the son is alive. Answer choice D is incorrect because the Rule Against Perpetuities does not require that a future interest become possessory within a life in being plus 21 years, but that it vest within that time period. Moreover, as the son was a life in being at the time of his father's death, it would be known within his lifetime whether he lives for 25 years after his father's death.

A privately owned architectural firm incorporated in State A contracted with the federal government to design and construct a federal research facility on federal land in State B. Because the firm wanted to source its construction materials close to the construction site, it purchased most of the needed supplies in State B from State B manufacturers. During the lengthy project, State B passed a law imposing a higher sales tax on industrial construction materials sold in State B. The architectural firm has challenged the constitutionality of the tax as applied to the supplies purchased for construction of the federal facility. Is the tax constitutional as applied to the supplies purchased by the firm for construction of the federal facility? A. No, because the federal government and its instrumentalities are immune from state taxation. B. No, because states cannot pass legislation retroactively impairing contractual rights and obligations. C. Yes, because the private firm is not immune from nondiscriminatory state taxation. D. Yes, because the state has the exclusive power to tax intrastate commerce.

Answer Choice C. The federal government and its instrumentalities are immune from taxation by the states. However, states may impose generally applicable indirect taxes so long as they do not unreasonably burden the federal government (e.g., state income taxes on federal employees). Here, this generally applicable sales tax on industrial construction materials that applies to a private architecture firm building a federal building is not so unreasonably burdensome as to be an impermissible taxation of the federal government. Answer choice A is incorrect because parties who contract to perform a service for the federal government are not immune from state taxation unless it is somehow so burdensome that it constitutes taxation of the government. Answer choice B is incorrect. State legislation that substantially impairs a contract between private parties is invalid, unless the government can demonstrate that the interference was reasonable and necessary to serve an important governmental interest. This legislation does not substantially impair a contractual right or duty; rather, it makes performance of a contracted duty more difficult, but not prohibitively so. Answer choice D is incorrect because Congress can tax and spend for the general welfare, and this can include taxation of intrastate commerce that has an aggregate effect on interstate commerce.

A state official was charged with two federal offenses—bribery in connection with a program administered by the state official that receives federal funds, and interstate travel for a criminal purpose. The bribe allegedly consisted of paying the expenses of an out-of-state trip taken by the state official in exchange for awarding a grant under the federal program to the payor of the expenses. The jury found the state official guilty of bribery but not guilty of the offense of interstate travel for a criminal purpose. The state official has challenged his conviction for bribery as unconstitutional. How should the court rule on this challenge? A. For the state official, because his conviction for bribery is inconsistent with acquittal of the interstate travel offense. B. For the state official, because the travel offense was a lesser-included offense of the bribery offense. C. Against the state official, because acquittal by a jury of an offense does not mandate acquittal of the predicate offense. D. Against the state official, because a jury verdict is not subject to challenge on appeal.

Answer Choice C. When a jury renders a verdict that a defendant is guilty of certain offenses but not guilty of other related offenses, the guilty verdict is not reviewable on the grounds of inconsistency, even when the jury acquits the defendant of an offense that is a predicated on an offense for which the same jury finds the defendant guilty. Consequently, although the defendant was acquitted by the jury of the travel offense which was predicated on bribery as its criminal purpose, the jury's verdict finding the defendant guilty of bribery is not reviewable on inconsistency grounds. Answer choice A is incorrect because, when there are inconsistent jury verdicts, due process does not mandate the overturning of the guilty verdict on the grounds that it is inconsistent with the acquittal. Answer choice B is incorrect because one of the offenses is not a lesser-included offense of the other offense; each requires proof of an element that the other does not. The travel offense requires proof that the travel involved interstate travel, while the bribery offense requires the solicitation or acceptance of something of value by a state or local official. Moreover, while the Double Jeopardy Clause may prevent a defendant from being punished for both a lesser- and a greater-included offense, it does not prevent a defendant from being tried for both offenses in the same trial, when the defendant is punished for only one of them. Here, the state official was only convicted of the bribery offense. Answer choice D is incorrect because a defendant may challenge a jury verdict on appeal on a number of grounds, including insufficiency of the evidence.

On May 1, a tourist went on a rock-climbing trip with a rock climbing guide. While on their climb, an anchor, used to attach a climber to the climbing surface, was placed on the cliff face by the guide. However, the anchor broke loose, and the guide fell, nearly taking the tourist with him. Luckily, the tourist caught a stone outcropping, saving both himself and the guide. The guide, fearing that the anchor had failed because he had placed it negligently, entered into a written contract with the tourist providing that the guide promised to pay the tourist $200,000 in consideration for saving the guide's life, and for the tourist's promise to not bring a negligence action against the guide for his injuries related to the fall. The contract provided that the guide would pay the money by August 1. Honestly believing he had a valid negligence claim against the guide, but unwilling to pay the legal costs of bringing such an action, the tourist signed the contract. A month later, an investigation revealed that the guide had not attached the anchor negligently, and that the failure was due to a manufacturing defect of the anchor. This discovery made it clear that in the applicable jurisdiction, the guide could not have been found liable to the tourist for negligence. On August 1, the tourist demanded payment under the written contract, but the guide refused to pay. The tourist sued the guide to recover $200,000 under the contract. Will the tourist succeed in his action? A. No, because past acts are typically insufficient consideration. B. No, because the tourist had no valid cause of action against the guide. C. Yes, because the contract was in writing and signed by the guide. D. Yes, because the tourist believed he had a valid negligence claim against the guide.

Answer Choice D. A promise not to bring a legal action or to assert a particular claim or defense in such an action generally can serve as consideration for a settlement agreement because the party making the promise is foregoing a legal right. The legal action need not be one that is certain to succeed. Instead, the claim or defense must be in fact doubtful due to uncertainty of facts or law, or the party failing to assert the claim or defense must believe in good faith that it may be fairly determined to be valid. Here, the tourist had a good-faith belief as to the validity of the negligence claim against the guide. Therefore, the contract between the guide and the tourist was supported by valid consideration and is enforceable against the guide. Answer choice A is incorrect. Even though the tourist's rescue of the guide would generally not constitute valid consideration because it was not bargained for, his promise not to bring a negligence action against the guide is valid consideration. Answer choice B is incorrect. As long as the party agreeing to forbear bringing the action had an honest belief as to the validity of the claim and a reasonable basis for that belief, that promise will constitute valid consideration. Answer choice C is incorrect because this contract is not subject to the Statute of Frauds. Therefore, the fact that it is contained in a signed writing is not determinative of its enforceability.

A man left a bag of cash with his sister. When he and his wife went to pick it up, the sister could not produce the bag. As the three of them stood in the sister's kitchen, the wife said to the man, "Tell her that the money in the bag came from the bank you robbed." The man said nothing, but before he left, he threatened to harm his sister if she did not bring him the bag before midnight. The man was charged with bank robbery. At his trial at which the he did not testify, the prosecution called the man's sister as a witness to testify as to what the man's wife had said to him while in the sister's kitchen. Is this testimony likely admissible? A. No, because of the spousal communication privilege. B. No, because the wife's statement is hearsay. C. Yes, because the man is unavailable to testify. D. Yes, because the statement was an adoptive admission.

Answer Choice D. An out-of-court statement offered for the truth of the matter asserted is generally hearsay. However, an out-of-court statement made by a party to the current litigation is admissible as "non-hearsay" if it is offered by an opposing party, even when it is offered for the truth of the matter asserted. Here, the statement was made out of court and is being offered for its truth—that the man robbed the bank. However, the statement was made by the man's wife, who is not a party, not the man himself. Silence in response to a statement is considered an adoptive admission if (i) the person was present and heard and understood the statement, (ii) the person had the ability and opportunity to deny the statement, and (iii) a reasonable person similarly situated would have denied the statement. Here, the man was present when his wife made the statement, and his subsequent threat to his sister before the man and his wife left his sister's apartment indicates that he heard and understood the statement and could have disavowed it if he had chosen to do so. Moreover, it is likely that a reasonable person would deny being a bank robber when being characterized as such. Answer choice A is incorrect because the spousal communication privilege does not apply to this statement. Although the statement was made by the man's wife to him, the presence of his sister when the statement was made prevents the man from asserting that the statement was made in confidence, which is a necessary element of this privilege. Answer choice B is incorrect because although this type of statement would typically constitute hearsay, it is admissible as an adoptive admission by an opposing party, and is thus "non-hearsay." Answer choice C is incorrect. Although the man as the defendant in a criminal trial was not compelled to testify, and having elected not to do so was thus unavailable as a witness, a statement of an opposing party is admissible regardless of whether the opposing party is available as a witness.

A girlfriend and boyfriend decided to rob a convenience store. They tied up the cashier and then raided the cash register, but it was empty, so the boyfriend decided to take the ATM in front of the store. The boyfriend dragged the ATM to his truck and loaded into the back. Back at the boyfriend's house, the boyfriend tried for many hours to crack open the ATM, but with no luck. The boyfriend decided to look for help on the Internet, and one blog suggested accessing the cash through a removable panel located on the bottom of the ATM. As the boyfriend leaned the ATM against a wall, it slipped, fell on top of him, and crushed him to death. Using parking lot security camera footage, the police were able to identify the boyfriend and girlfriend. When the police found the boyfriend's body, the girlfriend was apprehended and charged with felony murder. Which of the following, if established, would be the girlfriend's best defense? A. The girlfriend was not the principal of the underlying felony. B. The girlfriend did not act maliciously with regard to her boyfriend's death. C. The girlfriend did not act with a callous disregard for her boyfriend's life. D. The boyfriend's death was not causally connected to the girlfriend's felony.

Answer Choice D. Felony murder is an unintended killing proximately caused by and during the commission or attempted commission of an inherently dangerous felony. To defend against a felony murder charge successfully, the defendant can establish that the death was not a foreseeable result or a natural and probable consequence of the felony (i.e., there was no proximate causation). Here, the girlfriend's best defense is that the boyfriend's death was not proximately caused by the robbery. Although they stole the ATM from the convenience store that they attempted to rob, the boyfriend's death was not a natural or probable consequence of that robbery. He did not die during the commission of the felony, or while he was fleeing the scene of the crime, which would be foreseeable situations where the boyfriend might be killed; he died when the ATM fell on top of him as he was trying to crack it open in his home. Answer choice A is incorrect because there is no requirement that a defendant be the principal party to a crime to be liable for felony murder. Even if the girlfriend can successfully argue that she was only an accomplice with regards to the robbery, she would still be liable for the robbery and any other crimes if they were the natural and probable consequence of the accomplice's conduct. Because the cash register was empty, it is arguably natural and probable that the boyfriend and girlfriend might try to take the ATM instead. Thus, the girlfriend is probably liable for the underlying felony of robbery, even if as an accomplice. Answer choice B is incorrect because felony murder does not require the defendant to act maliciously in bringing about the death of the victim. In addition, the boyfriend was not a victim, but rather a co-felon. Answer choice C is incorrect because it states the standard for common-law murder, not felony murder.

A manufacturer of inflatable slides sold three of the slides to an indoor amusement park. The three slides were subjected to the manufacturer's usual rigorous inspection process. The slides were outsourced for delivery, and were delivered by a service in a standard delivery van. When the amusement park received the slides, the park's managers left the slides outside in the sun for a week before they were inflated and installed by an independent contractor. One week later, while a child was riding down one of the slides, the slide burst a seam and rapidly deflated. As a result, the child fell and broke his arm. The mother of the child sued the manufacturer of the slides under a negligence theory to recover for her child's injuries. Of the following, what is the manufacturer's best defense against the negligence charge? A. The child assumed the risk of being injured when he played on an inflatable slide. B. The slide was not under the exclusive control of the manufacturer before it ruptured. C. The mother and her child were not in privity with either the manufacturer or the park. D. The manufacturer exercised reasonable care in the inspection of these slides and discovered no defects.

Answer Choice D. In a products liability action based on negligence, defendants have a duty to exercise reasonable care in the inspection of a product. If a product is defective, the plaintiff must prove that the defendant would have discovered the defect if they had exercised reasonable care. In this case, the defendant subjected all of its slides—including these slides-to a rigorous inspection process. Accordingly, the defendant did not breach its duty to the plaintiff. Answer choice A is incorrect because regardless of how assumption of risk is treated in this jurisdiction, there is no evidence that the child voluntarily and unreasonably assumed the risk that the slide would rupture, causing him to fall. This kind of malfunction was not the kind of risk inherent in playing at an amusement park. Therefore, the child did not assume the risk of this injury. Answer choice B is incorrect. The "exclusive control" element is relevant to the doctrine of res ipsa loquitor, which allows the trier of fact to infer negligence in the part of the defendant in the absence of direct evidence. However, application of the doctrine isn't appropriate in all negligence cases, and there is no indication that the plaintiff is even depending on the doctrine in this case. A better defense is simply that the defendant did not act negligently. Answer choice C is incorrect because there is no privity requirement; a commercial manufacturer owes a duty to any foreseeable plaintiff, whether a purchaser, user, or bystander.

A local sporting goods store created an advertisement that aired only on local television. The advertisement featured a video of the goods offered by the store along with phrases regarding the quality of the products, such as, "Best in the Area!" One portion of the advertisement featured a display of the boxing equipment offered by the sporting goods store. At the same time, a voice said, "If you want to be the best, you have to train like the best." This slogan has been used by and is associated with a well-known professional boxer who grew up and continues to live in the area. The slogan appears on the boxer's boxing shorts, and the voice in the advertisement sounded very much like that of the boxer. The advertisement was made without the boxer's permission. When the boxer learned of the advertisement, he immediately brought an action against the sporting goods store for misappropriation of the right to publicity. He can establish that the advertisement has caused him the loss of endorsement income from another sporting goods store. Will the boxer prevail? A. No, because the advertisement did not use the boxer's name or picture. B. No, because the advertisement was only shown on local television. C. Yes, because the boxer can prove special damages. D. Yes, because the boxer did not consent to the advertisement.

Answer Choice D. Misappropriation of the right to publicity is based upon the right of an individual to control the commercial use of his own identity. The plaintiff must prove (i) the defendant's unauthorized appropriation of the plaintiff's name, likeness, or identity, (ii) for the defendant's advantage, commercial or otherwise, (iii) lack of consent, and (iv) resulting injury. Frequently, commercial appropriation cases involve the use of the plaintiff's name or picture, but this is not required. A television or radio production may simply mimic a plaintiff's distinctive vocal patterns in order to incur liability. In this case, the sporting goods store used the boxer's well-known slogan and mimicked his voice in their advertisement in order to gain more business. The advertisement was made without the boxer's consent. Therefore, the boxer will prevail in his action. Answer choice A is incorrect. The defendant need not use the plaintiff's name or picture; a television production mimicking the plaintiff's voice is sufficient for a claim of misappropriation of the right to publicity. Answer choice B is incorrect. Limited publication does not limit liability for misappropriation of the right to publicity. Answer choice C is incorrect. While the boxer's economic loss clearly qualifies as special damages, special damages are not required to be established in a misappropriation of the right to publicity action.

The driver of a car struck a pedestrian and then hit a utility pole. The driver was taken to an emergency room where he was treated by the attending physician for the resulting injuries to his arms and legs. The pedestrian later sued the driver based on negligence for damages stemming from her injuries. The pedestrian calls the attending physician to testify that the driver smelled of alcohol when he was brought into the emergency room. Applicable state law recognizes a physician-patient testimonial privilege applicable to statements made to all physicians. Is it likely that the driver can successfully assert the physician-patient privilege to prevent the physician from testifying? A. Yes, because the privilege applies to all physicians. B. Yes, because the driver was seeking treatment during his encounter with the physician. C. No, because the driver is the defendant in this action. D. No, because the testimony does not relate to a communication made by the driver to the physician.

Answer Choice D. Most states recognize a physician-patient privilege that protects communications made by a patient to a physician for the purpose of obtaining treatment. However, in this case, the testimony of the emergency room physician does not concern a communication made by the patient to the physician, but instead involves an observation by the physician of the patient's condition. Consequently, it is likely that the defendant-patient cannot prevent the physician from testifying as to as to his inebriated condition when he was brought to the emergency room. Answer choice A is incorrect because, even though the state recognizes a physician-patient privilege that extends to all physicians, the privilege likely does not apply to the physician's own observations made about the patient's condition. Answer choice B is incorrect because, although the interaction between the defendant-patient and the emergency room physician was for the purpose of treating his injuries, the privilege likely does not apply to the information about which the pedestrian seeks to have the emergency room physician testify (i.e., the defendant-patient's inebriated condition when he was brought to the emergency room). Answer choice C is incorrect because, even though the defendant-patient is not seeking to recover for these injuries, his physical condition is likely at issue in this action; his inebriated condition would be relevant in establishing his negligence in striking the pedestrian.

A building safety code mandated contractors who built homes in an area known for termite infestation to use termite-resistance woods, such as redwood or cedar, to form the framework of new homes. The safety code imposed a moderate fine on contractors who failed to obey this mandate. A contractor was hired by a woman to build a two-story home. Based upon availability and budget constraints, the contractor chose to use pine in order to build the home's framework. The contractor knew that pine was cheaper than and generally as sturdy as termite-resistant woods, even though it was the type of wood most prone to termites. One year after the home was complete, the second floor of the home collapsed because the pine framework was not durable enough to withstand the climate and temperature in the area. The woman suffered significant injuries in the collapse. If the woman sues the contractor under a negligence per se theory, is she likely to succeed? A. Yes, because the woman was a foreseeable plaintiff as her home was in the area covered by the building safety code. B. Yes, because the contractor violated the building safety code regarding the type of wood to use to build the framework for the home. C. No, because the contractor used reasonable care when choosing pine for the framework. D. No, because the collapse of the second floor due to the climate and temperature was not the type of harm the code was designed to prevent.

Answer Choice D. Negligence per se occurs when a defendant violates a criminal or regulatory statute that imposes a penalty for failure to meet a specific duty, and the defendant's failure to satisfy that duty caused the kind of harm the statute was designed to prevent. Here, the contractor violated the building safety code by using a type of wood that was not resistant to termites. However, the second floor of the home collapsed due to climate and temperature, not termite damage. Therefore, a negligence per se argument will not be successful in this case. Answer choice A is incorrect because, even though the woman was a foreseeable plaintiff, she did not suffer the kind of harm the building safety code was intended to prevent. Therefore, negligence per se will not apply. Answer choice B is incorrect because, even though the contractor did not use the kind of wood mandated by the statute, the type of resulting harm was not the kind of harm the building safety code was intended to prevent. Answer choice C is incorrect. It is a defense that the defendant exercised reasonable care in attempting to comply with the statute. Here, the contractor did not attempt to comply with the safety code because he willingly and knowingly chose a wood that was not resistant to termites.

A dog owner was walking her small dog on a leash in her urban neighborhood when a stranger approached her. The stranger demanded that the owner give him the leash or he would kick the dog until she did. The owner, fearful that the stranger would make good on his threat, surrendered the leash to the stranger. The stranger picked up the dog without harming it and ran away. The stranger intended to return the dog if the owner offered a reward. When the owner did so, an accomplice returned the dog to its owner. The stranger was subsequently arrested and charged with robbery. Of the following, which presents the stranger's best defense? A. The dog was not taken from the owner's person. B. The dog was unharmed. C. The stranger did not intend to permanently deprive the owner of her dog. D. The stranger did not threaten to harm the owner.

Answer Choice D. Robbery is larceny from the person or presence of the person by force or intimidation. The threat must be of immediate serious physical injury to the victim, a close family member, or other person present. A threat to injure the owner's dog (i.e., property), rather than the owner, is likely insufficient to constitute intimidation for purposes of robbery. Answer choice A is incorrect. Even if the taking of a dog being walked on a leash by its owner does not constitute the taking of personal property from the person of the owner, it clearly constitutes the taking of personal property from the presence of the owner. Answer choice B is incorrect because larceny, which itself is an element of robbery, requires the taking and carrying away of the personal property of another with the intent to permanently deprive that person of the property. There is no requirement that the property be destroyed or otherwise harmed or altered. Therefore, the fact that the dog was not harmed is not a defense to robbery. Answer choice C is incorrect because the taking of personal property with the intent to return it to the owner only in exchange for a reward is sufficient to constitute an intent to permanently deprive the owner of the property.

A city adopted an ordinance that prohibited any apartment building from housing more than one convicted felon on the premises. A convicted felon owned an apartment building in the city. He lived in one of the apartments in the building with his brother, who was also a convicted felon. The building owner received a citation from the city for violating the ordinance, and was ordered to pay a fine and bring the building into compliance. The building owner sued the city, arguing that the ordinance violated his constitutional rights. What is the best argument for striking down the ordinance? A. The ordinance violates the Privileges and Immunities Clause. B. The ordinance violates the Equal Protection Clause. C. The ordinance violates the Eighth Amendment. D. The ordinance violates the Due Process Clause.

Answer Choice D. The guarantee of substantive due process is based upon the idea that laws should be reasonable and not arbitrary, and ensures that a governmental action that infringes upon a fundamental right is generally subject to strict scrutiny. Related persons have a fundamental right to live together in a single household. In this case, the ordinance prevented the man from living with his brother. Thus, the man's best argument is that the ordinance, by prohibiting him from living with a family member, violates substantive due process. Answer choice A is incorrect because the Privileges and Immunities Clause, which prohibits a state from discriminating against the citizens of another state, is not implicated in this case. Answer choice B is incorrect because the law does not employ a suspect classification, and thus any equal protection claim would be subject to rational basis review. The man would have a difficult time showing that the ordinance did not satisfy this standard. Answer choice C is incorrect because state and local governments do not violate the Eighth Amendment simply by imposing special disabilities on convicted felons, especially when doing so is reasonably related to protecting public safety.

A woman hired an upholsterer to reupholster her couch for $1,000. The contract was executed on October 5, and the reupholstered couch was to be completed by November 5. On November 5, the upholsterer completed reupholstering the couch. The upholsterer sent the woman a bill on November 6 requesting payment of $1,000 due by November 20. On November 10, the woman sent a letter, which the upholsterer received the next day. In the letter, the woman asserted in good faith that the couch had not been properly reupholstered, but that if the upholsterer properly upholstered the couch as he was required to do under their October 5 agreement, she would pay him $900. The upholsterer did not respond. The woman then mailed him a check for $900 with a notation that stated that it was for payment in full pursuant to the November 10 letter. The upholsterer received the check on November 18 and immediately cashed it, but did not reupholster the woman's couch. Can the woman sue the upholsterer for breach of the October 5 agreement? A. No, because the upholsterer did not accept the November 10 letter which was a counteroffer. B. No, because the upholsterer, by cashing the check, accepted the woman's November 10 offer, and entered into a new agreement with the woman. C. Yes, because the woman did not provide any consideration to support the accord agreement she offered in her November 10 letter. D. Yes, because the upholsterer had a preexisting duty to fulfill the October 5 contract.

Answer Choice D. The upholsterer was under a preexisting duty to fulfill the original contract executed on October 5, when he, by cashing the woman's check, entered into an accord agreement under which he impliedly promised to upholster the couch again. His acceptance of the woman's offer of an accord agreement did not discharge the original contract, but suspended it for a reasonable time to allow him to "satisfy" the accord agreement by reupholstering the coach. Because the upholsterer did not "satisfy" the accord, the woman can sue on their original contract. (Note: The upholsterer was not compelled to enter into the accord agreement. He could have refused to cash the check and sought a breach of contract claim against the woman. However, the upholsterer cashed the check on the condition that he upholster the couch again.) Answer choice A is incorrect because it misstates the validity of the original contract. A contract was already in place on November 10, and thus there was no outstanding offer to which the woman's letter served as a counter-offer. Answer choice B is incorrect. Although the upholsterer, by cashing the woman's check, did enter into an accord agreement with the woman, this accord agreement did not rescind the October 5 agreement, but merely suspended it. Answer choice C is incorrect because, with regard to the accord agreement, the woman provided consideration in the form of a promise to forego her rights to sue under the October 5 agreement. However, as noted with regard to answer choice D, the accord agreement did not rescind the October 5 agreement, but merely suspended it.

A woman hired her brother to dress up as a clown at her son's fifth birthday party. The woman told her brother that if he would dress up as a clown and entertain the children for two hours, she would pay off a $400 debt that he owed to his ex-girlfriend. The brother agreed, and showed up at the son's birthday party dressed as a clown. However, because he scared all of the children at the party, the woman refused to pay the ex-girlfriend $400, despite her demands for payment of the $400 debt. Finally, the woman offered to give the ex-girlfriend a gold necklace, which was worth approximately $250, as payment. The ex-girlfriend accepted the necklace, but then filed suit against the woman for $150. Will the ex-girlfriend succeed? A. Yes, because the gold necklace was only worth $250. B. Yes, because the ex-girlfriend was a third party beneficiary of the woman's contract with her brother. C. No, because the original agreement between the woman and her brother was not valid. D. No, because the ex-girlfriend accepted the gold necklace.

Answer Choice D. Under an accord agreement, a party to a contract agrees to accept a performance from the other party that differs from the performance that was promised in the existing contract, in satisfaction of the other party's existing duty. When a party agrees to accept a lesser amount in full satisfaction of its monetary claim, there must be consideration or a consideration substitute for the party's promise to accept the lesser amount. For example, consideration can exist if the other party honestly disputes the claim or agrees to forego an asserted defense, or if the payment is of a different type than called for under the original contract. Here, the original debt of $400 was discharged because the woman accepted a gold necklace, which was a different type of payment than what was called for under the agreement between the brother and the woman. For this reason, answer choice A is incorrect. Answer choice B is incorrect. The ex-girlfriend was an intended third-party beneficiary of the contract between the woman and her brother, and was therefore owed $400 under that contract. However, the ex-girlfriend's acceptance of the necklace as payment instead of $400 discharged the woman's obligation under the contract. Answer choice C is incorrect because the woman and the brother entered into a valid and enforceable contract for the brother to dress up as a clown at her son's birthday party in exchange for the woman's promise to pay the brother's $400 debt to his ex-girlfriend.

A state governmental water and sewer authority was sued in federal district court on the basis of diversity jurisdiction by a private contractor for an alleged breach of contract and the resulting damages. The authority, in answering the complaint, contended that it was immune from the suit under the Eleventh Amendment of the U.S. Constitution and moved to dismiss the action. The district court denied the motion. The authority filed an appeal of the district court's order denying its motion. The contractor has challenged the authority's right to appeal the district court's order. Should the appellate court permit this appeal? A. No, because of the final judgment rule. B. No, because the district court order denied the authority's motion to dismiss. C. Yes, because a constitutional right is at stake. D. Yes, under the collateral-order doctrine.

Answer Choice D. Under the collateral-order doctrine, a court of appeals has discretion to hear and rule on a district court order if it (i) conclusively determines the disputed question, (ii) resolves an important issue that is completely separate from the merits of the action, and (iii) is effectively unreviewable on appeal from a final judgment. A grant of immunity from prosecution, such as the immunity provided by the Eleventh Amendment, can fall within the narrow confines of the collateral-order doctrine because the benefit of immunity would be lost if the party claiming it were wrongfully forced to proceed to trial. Answer choice A is incorrect. Even though the district court order is not a final judgment, the collateral-order doctrine permits the appeal of this order. Answer choice B is incorrect. Although the denial of a motion to dismiss is not a final judgment and therefore not immediately appealable, the collateral-order doctrine permits the appeal of this order. Answer choice C is incorrect. First, governments do not technically have "rights"; rather, sovereign immunity is a specific constitutional rule that provides the government with a defense against come civil suits. Further, there is no broad exception to the final judgment rule based on the fact that a district court order affects a constitutional right.

A man was on trial for bank robbery and felony murder. The prosecution alleged that while the man went into a bank wearing a ski mask and wielding a gun, his getaway driver waited in a car outside. The man shot a bank patron during the robbery then got into the getaway car with the money. In the high-speed chase that followed, the getaway car crashed and the getaway driver died. At trial, the defense called a bartender who talked to the getaway driver the night before the robbery. The bartender testified that the getaway driver had told him that because the man refused to join him in the robbery unless they used a fake gun, the getaway driver had given him a real gun to use and told him it was fake. The court ruled that the statement was admissible as a statement against the getaway driver's interest. The prosecution now wishes to call a woman to testify that the getaway driver had told her a week before the robbery that the man had purchased a gun and was planning on robbing a bank. Is the woman's testimony admissible? A. No, because the getaway driver had no opportunity to explain or deny the statement. B. No, because the getaway driver's statement was not made under oath. C. Yes, under a hearsay exception for an unavailable declarant. D. Yes, to impeach the getaway driver.

Answer Choice D. When a hearsay statement is admitted into evidence, the credibility of the declarant may be attacked (and, if attacked, supported) by any evidence that would be admissible if the declarant had testified as a witness. Here, if the getaway driver had testified in court that he had given the man a fake gun, the woman would have been able to offer testimony of his prior inconsistent statement to impeach him. Therefore, the woman's testimony is proper impeachment evidence of a hearsay declarant, and is admissible for that purpose. Answer choice A is incorrect. When a hearsay declarant is impeached, the declarant need not be given the opportunity to explain or deny any inconsistent statement or conduct. Answer choice B is incorrect. Although an inconsistent statement that was not made under penalty of perjury is not admissible as substantive evidence, it may be admissible to impeach a witness. Therefore, the getaway driver's statement to the woman can still be admitted to impeach the getaway driver. Answer choice C is incorrect. There are five exceptions to the hearsay rule that apply only if the declarant is unavailable as a witness: former testimony, dying declaration, statement against interest, statement of personal or family history, and statement offered against a party that wrongfully caused the declarant's unavailability. The only exception that could arguably apply to the woman's testimony would be a statement against interest. However, under this exception to the hearsay rule, the statement must have been against the declarant's proprietary or pecuniary interest, have invalidated the declarant's claim against someone, or have exposed the declarant to civil or criminal liability. The getaway driver's statement to the woman does not fall within this exception because the statement only concerned the conduct of the man and did not implicate any interest of the getaway driver or expose him to criminal liability. Therefore, the statement is only admissible as impeachment evidence as a prior inconsistent statement, not as an exception to the hearsay rule.

A landowner entered into a written contract to sell five acres of land to a buyer for $50,000. The buyer agreed to pay for the land in installments, with $10,000 due on the date on which the contract is executed, and $10,000 due each year thereafter on the anniversary of that date. The contract was silent as to the date on which the landowner is to deliver the deed to the buyer. On what date must the landowner deliver the deed to the five acres of land to the buyer? A. When the contract is executed, because the buyer is obligated to act in good faith with regard to the payment of the remaining installments. B. The third anniversary of the execution of the contract, when the buyer has substantially performed. C. The fourth anniversary of the execution of the contract, because of the perfect tender rule. D. The fourth anniversary of the execution of the contract, when the buyer tenders the last installment payment.

Answer Choice D. When only one party's performance of his contractual duty requires a period of time, that party must complete his performance before the other party is required to perform, unless the language or circumstances indicate otherwise. Here, because the buyer is obligated to make five payments of $10,000 each, the buyer must tender the final payment in order for the landowner to be obligated to deliver the deed to the five acres of land to the buyer. Answer choice A is incorrect. Although a party to any contract is required to act in good faith, this requirement does not dictate the time at which the other party to the contract must perform. A party acting in good faith who nevertheless fails to perform a duty is not entitled to the other party's performance, despite acting in good faith. Answer choice B is incorrect because, although arguably the buyer has substantially performed his contractual obligation by paying the landowner 80 percent of the purchase price at that time, the buyer must pay the final installment of $10,000, or at least tender that amount, before the landowner is obligated to deliver the deed to the property. Answer choice C is incorrect because the perfect tender rule generally applies to a contract for the sale of goods, but does not apply to contracts governed by the common law, such as this contract, which is a contract for the sale of land.

Concerned with the proliferation of unwanted commercial emails, a state legislature enacted and the governor signed into law the Commercial Electronic Email Act. Under this Act, an in-state recipient of an unsolicited commercial email (i.e., "spam") that uses false or misleading information in the subject line of the email is entitled to at least $500 in damages from the sender of the email. In-state recipients of spam sent by a corporation incorporated and operating in a neighboring state filed a class action lawsuit against the sender for violation of the Act. The corporation that sent these emails has defended the action by contending that the Act violates the Dormant Commerce Clause. Which of the following would be the state's least effective argument to defend the constitutionality of the Act? A. You Selected: The Act does not discriminate against out-of-state commerce B. The Act does not place an undue burden on interstate commerce C. The Act does not regulate conduct that occurs wholly beyond the state's borders. D. The defendant-sender is a corporation and is not protected by the Dormant Commerce Clause.

Answer Choice D. While the Comity Clause of Article IV, Section 2 of the United States Constitution limits its protection to individuals, the Dormant Commerce Clause is not so limited. State or local action that discriminates against a corporation can violate the Dormant Commerce Clause. Consequently, the fact that the defendant-sender is a corporation would not be a defense against the corporation's challenge to the Act. Answer choice A is incorrect because a state can violate the Dormant Commerce Clause by discriminating against out-of-state commerce. Consequently, because the plaintiffs can show that this Act applies to all emails received by state residents regardless of the location of the sender, this argument could help defend against the defendant-sender's assertion that the Act violates the Dormant Commerce Clause. Answer choice B is incorrect because a state can violate the Dormant Commerce Clause by placing an undue burden on interstate commerce. Therefore, if the plaintiffs can show that this Act does not place an undue burden on interstate commerce, then this evidence would help defend against the defendant-sender's assertion of unconstitutionality. Answer choice C is incorrect because a state can violate the Dormant Commerce Clause by regulating conduct that occurs wholly beyond the state's borders. Because the plaintiffs can show that this Act only regulates emails sent to an in-state recipient, then this argument could help defend against the defendant-sender's assertion that the Act violates the Dormant Commerce Clause.

After being fired from his job, the defendant drank almost a quart of vodka and decided to ride the bus home. While on the bus, he saw a briefcase he mistakenly thought was his own, and began struggling with the passenger carrying the briefcase. The defendant knocked the passenger to the floor, took the briefcase, and fled. The defendant was arrested and charged with robbery. The defendant should be (A) acquitted, because he used no threats and was intoxicated. (B) acquitted, because his mistake negated the required specific intent. (C) convicted, because his intoxication was voluntary. (D) convicted, because his mistake is no defense to robbery.

Answer choice B is the best response. Because the defendant's mistaken belief that the briefcase was his own prevented him from having the required mental state for robbery. Robbery is the: (1) taking, (2) of personal property of another, (3) from the other's person or presence, (4) by force or intimidation, (5) with the intent to permanently deprive him of it. The intent required for robbery is the intent to use force to take the "property of another." Therefore, if the defendant mistakenly (even unreasonably) believes that the property in question is how, the required "intent to take the property of another" is lacking. The fact that the mistake was brought about by voluntary intoxication -- and the fact that the mistake may have been unreasonable -- makes no difference.

A defendant watched a liquor store furtively for some time, planning to hold it up. He bought a realistic-looking toy gun for the job. One night, just before the store's closing time, he drove to the store, opened the front door and entered. He reached in his pocket for the toy gun, but he became frightened and began to move back toward the front door. However, the shopkeeper had seen the butt of the gun. Fearing a holdup, the shopkeeper produced a gun from under the counter, pointed it at the defendant, and yelled, "Stop!" The defendant ran to the door and the toy gun fell from his pocket. The shopkeeper fired. The shot missed the defendant, but struck and killed a passerby outside the store. A statute in the jurisdiction defines burglary as "breaking and entering any building or structure with the intent to comment a felony or to steal therein." On a charge of burglary, the defendant's best defense would be that (A) the intent required was not present. (B) the liquor store was open to the public. (C) he had a change of heart and withdrew before committing any crime inside the store. (D) He was unsuccessful, and so at most he could only be guilty of attempted burglary.

Answer choice B. Because answer choice B is plausible under these facts, and negates one of the elements of burglary. Under the statute, burglary requires breaking and entering any building or structure with the intent to commit a felony or to steal therein. However, a structure that is open to the public cannot be "broken and entered," at least as to areas where, and times when, the public is welcome. Since the liquor store was open to the public, and the defendant entered it during its business hours, he could not satisfy the "breaking and entering" element of burglar. Since (B) cites a fact that will exonerate the defendant, and it fits these facts, it is the best response.


Set pelajaran terkait

Chapter 14: Nursing Management During Labor and Birth

View Set

Interpersonal Relationships chapter 10

View Set

chapter 15 Bio: evolution on a small scale

View Set

Module 7 Social Inequalities Exam

View Set